Search Results

Search found 636 results on 26 pages for 'serializable'.

Page 25/26 | < Previous Page | 21 22 23 24 25 26  | Next Page >

  • Grails, app-engine, jpa - beginner having trouble with grails generate-all

    - by John
    I'm trying to learn about grails with Google App Engine and JPA by following a few tutorials: http://www.morkeleb.com/2009/08/12/grails-and-google-appengine-beginners-guide/ http://inhouse32.appspot.com/index.html http://grails.org/plugin/app-engine I've got grails 1.3.0 RC 2, and App Engine SDK 1.3.3, and I'm using Windows 7. The steps that I try are: grails create-app appname cd appname grails install-plugin app-engine. I answer jpa when asked about jdo/jpa. It appears to install the gorm-jpa plugin automatically, although the tutorials all suggest installing gorm-jpa manually. grails install-plugin gorm-jpa (just in case) grails create-domain-class test.Person Edit the grails-app/domain/test/Person.groovy to add name and address fields: package test import javax.persistence.*; // import com.google.appengine.api.datastore.Key; @Entity class Person implements Serializable { @Id @GeneratedValue(strategy = GenerationType.IDENTITY) Long id @Basic String name @Basic String address static constraints = { id visible:false } } grails generate-all test.Person I get errors during this final step: C:\Users\John\Workspaces\STS\appname>grails generate-all test.Person Welcome to Grails 1.3.0.RC2 - http://grails.org/ Licensed under Apache Standard License 2.0 Grails home is set to: C:\Users\John\Downloads\grails-1.3.0.RC2\grails-1.3.0.RC2 Base Directory: C:\Users\John\Workspaces\STS\appname Resolving dependencies... Dependencies resolved in 493ms. Running script C:\Users\John\Downloads\grails-1.3.0.RC2\grails-1.3.0.RC2\scripts\GenerateAll.groovy Environment set to development [copy] Copied 4 empty directories to 2 empty directories under C:\Users\John\.grails\1.3.0.RC2\projects\appname\resources [copy] Copied 4 empty directories to 2 empty directories under C:\Users\John\.grails\1.3.0.RC2\projects\appname\resources [copy] Copied 1 empty directory to 1 empty directory under C:\Users\John\.grails\1.3.0.RC2\projects\appname\resources [mkdir] Created dir: C:\Users\John\Workspaces\STS\appname\web-app\WEB-INF\classes [groovyc] Compiling 12 source files to C:\Users\John\Workspaces\STS\appname\web-app\WEB-INF\classes Note: C:\Users\John\.grails\1.3.0.RC2\projects\appname\plugins\gorm-jpa-0.7.1\src\java\org\grails\jpa\domain\JpaGrailsDomainClass.java uses or overrides a deprecated API. Note: Recompile with -Xlint:deprecation for details. Note: Some input files use unchecked or unsafe operations. Note: Recompile with -Xlint:unchecked for details. [groovyc] Compiling 8 source files to C:\Users\John\Workspaces\STS\appname\web-app\WEB-INF\classes [mkdir] Created dir: C:\Users\John\.grails\1.3.0.RC2\projects\appname\resources\grails-app\i18n [native2ascii] Converting 13 files from C:\Users\John\Workspaces\STS\appname\grails-app\i18n to C:\Users\John\.grails\1.3.0.RC2\projects\appname\resources\grails-app\i18n [mkdir] Created dir: C:\Users\John\.grails\1.3.0.RC2\projects\appname\resources\plugins\gorm-jpa-0.7.1\grails-app\i18n [mkdir] Created dir: C:\Users\John\.grails\1.3.0.RC2\projects\appname\resources\plugins\app-engine-0.8.10\grails-app\i18n [native2ascii] Converting 1 file from C:\Users\John\.grails\1.3.0.RC2\projects\appname\plugins\gorm-jpa-0.7.1\grails-app\i18n to C:\Users\John\.grails\1.3.0.RC2\projects\appname\resources\plugins\gorm -jpa-0.7.1\grails-app\i18n [native2ascii] Converting 1 file from C:\Users\John\.grails\1.3.0.RC2\projects\appname\plugins\app-engine-0.8.10\grails-app\i18n to C:\Users\John\.grails\1.3.0.RC2\projects\appname\resources\plugins\a pp-engine-0.8.10\grails-app\i18n [copy] Copying 1 file to C:\Users\John\Workspaces\STS\appname\web-app\WEB-INF\classes [copy] Copying 2 files to C:\Users\John\.grails\1.3.0.RC2\projects\appname\resources [copy] Copied 2 empty directories to 2 empty directories under C:\Users\John\.grails\1.3.0.RC2\projects\appname\resources [copy] Copying 1 file to C:\Users\John\.grails\1.3.0.RC2\projects\appname [mkdir] Created dir: C:\Users\John\Workspaces\STS\appname\web-app\plugins\app-engine-0.8.10 [copy] Copying 1 file to C:\Users\John\Workspaces\STS\appname\web-app\plugins\app-engine-0.8.10 [copy] Copying 1 file to C:\Users\John\Workspaces\STS\appname\web-app\WEB-INF [mkdir] Created dir: C:\Users\John\Workspaces\STS\appname\web-app\WEB-INF\lib [copy] Copying 64 files to C:\Users\John\Workspaces\STS\appname\web-app\WEB-INF\lib Configuring persistence for AppEngine [mkdir] Created dir: C:\Users\John\Workspaces\STS\appname\web-app\WEB-INF\classes\META-INF [copy] Copying 1 file to C:\Users\John\Workspaces\STS\appname\web-app\WEB-INF\classes\META-INF [mkdir] Created dir: C:\Users\John\Workspaces\STS\appname\web-app\WEB-INF\plugins\app-engine-0.8.10 [copy] Copying 2 files to C:\Users\John\Workspaces\STS\appname\web-app\WEB-INF\plugins\app-engine-0.8.10 [mkdir] Created dir: C:\Users\John\Workspaces\STS\appname\web-app\WEB-INF\plugins\gorm-jpa-0.7.1 [copy] Copying 2 files to C:\Users\John\Workspaces\STS\appname\web-app\WEB-INF\plugins\gorm-jpa-0.7.1 Packaging AppEngine jar files Enhancing JDO classes [enhance] DataNucleus Enhancer (version 1.1.4) : Enhancement of classes [enhance] DataNucleus Enhancer completed with success for 1 classes. Timings : input=589 ms, enhance=200 ms, total=789 ms. Consult the log for full details [groovyc] Compiling 1 source file to C:\Users\John\Workspaces\STS\appname\web-app\WEB-INF\classes [copy] Copying 1 file to C:\Users\John\.grails\1.3.0.RC2\projects\appname [copy] Copying 1 file to C:\Users\John\Workspaces\STS\appname\web-app\WEB-INF Configuring persistence for AppEngine Packaging AppEngine jar files Enhancing JDO classes [enhance] DataNucleus Enhancer (version 1.1.4) : Enhancement of classes [enhance] DataNucleus Enhancer completed with success for 1 classes. Timings : input=585 ms, enhance=28 ms, total=613 ms. Consult the log for full details Generating views for domain class test.Person ... java.lang.reflect.InvocationTargetException at SimpleTemplateScript1.run(SimpleTemplateScript1.groovy:43) at _GrailsGenerate_groovy.generateForDomainClass(_GrailsGenerate_groovy:85) at _GrailsGenerate_groovy$_run_closure1.doCall(_GrailsGenerate_groovy:50) at GenerateAll$_run_closure1.doCall(GenerateAll.groovy:42) at gant.Gant$_dispatch_closure5.doCall(Gant.groovy:381) at gant.Gant$_dispatch_closure7.doCall(Gant.groovy:415) at gant.Gant$_dispatch_closure7.doCall(Gant.groovy) at gant.Gant.withBuildListeners(Gant.groovy:427) at gant.Gant.this$2$withBuildListeners(Gant.groovy) at gant.Gant$this$2$withBuildListeners.callCurrent(Unknown Source) at gant.Gant.dispatch(Gant.groovy:415) at gant.Gant.this$2$dispatch(Gant.groovy) at gant.Gant.invokeMethod(Gant.groovy) at gant.Gant.executeTargets(Gant.groovy:590) at gant.Gant.executeTargets(Gant.groovy:589) Caused by: java.lang.NoClassDefFoundError: org/hibernate/MappingException ... 15 more Caused by: java.lang.ClassNotFoundException: org.hibernate.MappingException at org.codehaus.groovy.tools.RootLoader.findClass(RootLoader.java:156) at java.lang.ClassLoader.loadClass(ClassLoader.java:307) at org.codehaus.groovy.tools.RootLoader.loadClass(RootLoader.java:128) at java.lang.ClassLoader.loadClass(ClassLoader.java:248) ... 15 more Error running generate-all: null What am I doing wrong?

    Read the article

  • Grails, app-engine, jpa - TargetInvocationException

    - by John
    I'm trying to learn about grails with Google App Engine and JPA by following a few tutorials: http://www.morkeleb.com/2009/08/12/grails-and-google-appengine-beginners-guide/ http://inhouse32.appspot.com/index.html http://grails.org/plugin/app-engine I've got grails 1.3.0 RC 2, and App Engine SDK 1.3.3, and I'm using Windows 7. The steps that I try are: grails create-app appname cd appname grails install-plugin app-engine. I answer jpa when asked about jdo/jpa. It appears to install the gorm-jpa plugin automatically, although the tutorials all suggest installing gorm-jpa manually. grails install-plugin gorm-jpa (just in case) grails create-domain-class test.Person Edit the grails-app/domain/test/Person.groovy to add name and address fields: package test import javax.persistence.*; // import com.google.appengine.api.datastore.Key; @Entity class Person implements Serializable { @Id @GeneratedValue(strategy = GenerationType.IDENTITY) Long id @Basic String name @Basic String address static constraints = { id visible:false } } grails generate-all test.Person I get errors during this final step: C:\Users\John\Workspaces\STS\appname>grails generate-all test.Person Welcome to Grails 1.3.0.RC2 - http://grails.org/ Licensed under Apache Standard License 2.0 Grails home is set to: C:\Users\John\Downloads\grails-1.3.0.RC2\grails-1.3.0.RC2 Base Directory: C:\Users\John\Workspaces\STS\appname Resolving dependencies... Dependencies resolved in 493ms. Running script C:\Users\John\Downloads\grails-1.3.0.RC2\grails-1.3.0.RC2\scripts\GenerateAll.groovy Environment set to development [copy] Copied 4 empty directories to 2 empty directories under C:\Users\John\.grails\1.3.0.RC2\projects\appname\resources [copy] Copied 4 empty directories to 2 empty directories under C:\Users\John\.grails\1.3.0.RC2\projects\appname\resources [copy] Copied 1 empty directory to 1 empty directory under C:\Users\John\.grails\1.3.0.RC2\projects\appname\resources [mkdir] Created dir: C:\Users\John\Workspaces\STS\appname\web-app\WEB-INF\classes [groovyc] Compiling 12 source files to C:\Users\John\Workspaces\STS\appname\web-app\WEB-INF\classes Note: C:\Users\John\.grails\1.3.0.RC2\projects\appname\plugins\gorm-jpa-0.7.1\src\java\org\grails\jpa\domain\JpaGrailsDomainClass.java uses or overrides a deprecated API. Note: Recompile with -Xlint:deprecation for details. Note: Some input files use unchecked or unsafe operations. Note: Recompile with -Xlint:unchecked for details. [groovyc] Compiling 8 source files to C:\Users\John\Workspaces\STS\appname\web-app\WEB-INF\classes [mkdir] Created dir: C:\Users\John\.grails\1.3.0.RC2\projects\appname\resources\grails-app\i18n [native2ascii] Converting 13 files from C:\Users\John\Workspaces\STS\appname\grails-app\i18n to C:\Users\John\.grails\1.3.0.RC2\projects\appname\resources\grails-app\i18n [mkdir] Created dir: C:\Users\John\.grails\1.3.0.RC2\projects\appname\resources\plugins\gorm-jpa-0.7.1\grails-app\i18n [mkdir] Created dir: C:\Users\John\.grails\1.3.0.RC2\projects\appname\resources\plugins\app-engine-0.8.10\grails-app\i18n [native2ascii] Converting 1 file from C:\Users\John\.grails\1.3.0.RC2\projects\appname\plugins\gorm-jpa-0.7.1\grails-app\i18n to C:\Users\John\.grails\1.3.0.RC2\projects\appname\resources\plugins\gorm -jpa-0.7.1\grails-app\i18n [native2ascii] Converting 1 file from C:\Users\John\.grails\1.3.0.RC2\projects\appname\plugins\app-engine-0.8.10\grails-app\i18n to C:\Users\John\.grails\1.3.0.RC2\projects\appname\resources\plugins\a pp-engine-0.8.10\grails-app\i18n [copy] Copying 1 file to C:\Users\John\Workspaces\STS\appname\web-app\WEB-INF\classes [copy] Copying 2 files to C:\Users\John\.grails\1.3.0.RC2\projects\appname\resources [copy] Copied 2 empty directories to 2 empty directories under C:\Users\John\.grails\1.3.0.RC2\projects\appname\resources [copy] Copying 1 file to C:\Users\John\.grails\1.3.0.RC2\projects\appname [mkdir] Created dir: C:\Users\John\Workspaces\STS\appname\web-app\plugins\app-engine-0.8.10 [copy] Copying 1 file to C:\Users\John\Workspaces\STS\appname\web-app\plugins\app-engine-0.8.10 [copy] Copying 1 file to C:\Users\John\Workspaces\STS\appname\web-app\WEB-INF [mkdir] Created dir: C:\Users\John\Workspaces\STS\appname\web-app\WEB-INF\lib [copy] Copying 64 files to C:\Users\John\Workspaces\STS\appname\web-app\WEB-INF\lib Configuring persistence for AppEngine [mkdir] Created dir: C:\Users\John\Workspaces\STS\appname\web-app\WEB-INF\classes\META-INF [copy] Copying 1 file to C:\Users\John\Workspaces\STS\appname\web-app\WEB-INF\classes\META-INF [mkdir] Created dir: C:\Users\John\Workspaces\STS\appname\web-app\WEB-INF\plugins\app-engine-0.8.10 [copy] Copying 2 files to C:\Users\John\Workspaces\STS\appname\web-app\WEB-INF\plugins\app-engine-0.8.10 [mkdir] Created dir: C:\Users\John\Workspaces\STS\appname\web-app\WEB-INF\plugins\gorm-jpa-0.7.1 [copy] Copying 2 files to C:\Users\John\Workspaces\STS\appname\web-app\WEB-INF\plugins\gorm-jpa-0.7.1 Packaging AppEngine jar files Enhancing JDO classes [enhance] DataNucleus Enhancer (version 1.1.4) : Enhancement of classes [enhance] DataNucleus Enhancer completed with success for 1 classes. Timings : input=589 ms, enhance=200 ms, total=789 ms. Consult the log for full details [groovyc] Compiling 1 source file to C:\Users\John\Workspaces\STS\appname\web-app\WEB-INF\classes [copy] Copying 1 file to C:\Users\John\.grails\1.3.0.RC2\projects\appname [copy] Copying 1 file to C:\Users\John\Workspaces\STS\appname\web-app\WEB-INF Configuring persistence for AppEngine Packaging AppEngine jar files Enhancing JDO classes [enhance] DataNucleus Enhancer (version 1.1.4) : Enhancement of classes [enhance] DataNucleus Enhancer completed with success for 1 classes. Timings : input=585 ms, enhance=28 ms, total=613 ms. Consult the log for full details Generating views for domain class test.Person ... java.lang.reflect.InvocationTargetException at SimpleTemplateScript1.run(SimpleTemplateScript1.groovy:43) at _GrailsGenerate_groovy.generateForDomainClass(_GrailsGenerate_groovy:85) at _GrailsGenerate_groovy$_run_closure1.doCall(_GrailsGenerate_groovy:50) at GenerateAll$_run_closure1.doCall(GenerateAll.groovy:42) at gant.Gant$_dispatch_closure5.doCall(Gant.groovy:381) at gant.Gant$_dispatch_closure7.doCall(Gant.groovy:415) at gant.Gant$_dispatch_closure7.doCall(Gant.groovy) at gant.Gant.withBuildListeners(Gant.groovy:427) at gant.Gant.this$2$withBuildListeners(Gant.groovy) at gant.Gant$this$2$withBuildListeners.callCurrent(Unknown Source) at gant.Gant.dispatch(Gant.groovy:415) at gant.Gant.this$2$dispatch(Gant.groovy) at gant.Gant.invokeMethod(Gant.groovy) at gant.Gant.executeTargets(Gant.groovy:590) at gant.Gant.executeTargets(Gant.groovy:589) Caused by: java.lang.NoClassDefFoundError: org/hibernate/MappingException ... 15 more Caused by: java.lang.ClassNotFoundException: org.hibernate.MappingException at org.codehaus.groovy.tools.RootLoader.findClass(RootLoader.java:156) at java.lang.ClassLoader.loadClass(ClassLoader.java:307) at org.codehaus.groovy.tools.RootLoader.loadClass(RootLoader.java:128) at java.lang.ClassLoader.loadClass(ClassLoader.java:248) ... 15 more Error running generate-all: null What am I doing wrong?

    Read the article

  • Very basic running of drools 5, basic setup and quickstart

    - by Berlin Brown
    Is there a more comprehensive quick start for drools 5. I was attempting to run the simple Hello World .drl rule but I wanted to do it through an ant script, possibly with just javac/java: I get the following error: Note: I don't am running completely without Eclipse or any other IDE: Is there a more comprehensive quick start for drools 5. I was attempting to run the simple Hello World .drl rule but I wanted to do it through an ant script, possibly with just javac/java: I get the following error: Note: I don't am running completely without Eclipse or any other IDE: test: [java] Exception in thread "main" org.drools.RuntimeDroolsException: Unable to load d ialect 'org.drools.rule.builder.dialect.java.JavaDialectConfiguration:java:org.drools.rule .builder.dialect.java.JavaDialectConfiguration' [java] at org.drools.compiler.PackageBuilderConfiguration.addDialect(PackageBuild erConfiguration.java:274) [java] at org.drools.compiler.PackageBuilderConfiguration.buildDialectConfigurati onMap(PackageBuilderConfiguration.java:259) [java] at org.drools.compiler.PackageBuilderConfiguration.init(PackageBuilderConf iguration.java:176) [java] at org.drools.compiler.PackageBuilderConfiguration.<init>(PackageBuilderCo nfiguration.java:153) [java] at org.drools.compiler.PackageBuilder.<init>(PackageBuilder.java:242) [java] at org.drools.compiler.PackageBuilder.<init>(PackageBuilder.java:142) [java] at org.drools.builder.impl.KnowledgeBuilderProviderImpl.newKnowledgeBuilde r(KnowledgeBuilderProviderImpl.java:29) [java] at org.drools.builder.KnowledgeBuilderFactory.newKnowledgeBuilder(Knowledg eBuilderFactory.java:29) [java] at org.berlin.rpg.rules.Rules.rules(Rules.java:33) [java] at org.berlin.rpg.rules.Rules.main(Rules.java:73) [java] Caused by: java.lang.RuntimeException: The Eclipse JDT Core jar is not in the classpath [java] at org.drools.rule.builder.dialect.java.JavaDialectConfiguration.setCompil er(JavaDialectConfiguration.java:94) [java] at org.drools.rule.builder.dialect.java.JavaDialectConfiguration.init(Java DialectConfiguration.java:55) [java] at org.drools.compiler.PackageBuilderConfiguration.addDialect(PackageBuild erConfiguration.java:270) [java] ... 9 more [java] Java Result: 1 ... ... I do include the following libraries with my javac and java target: <path id="classpath"> <pathelement location="${lib.dir}" /> <pathelement location="${lib.dir}/drools-api-5.0.1.jar" /> <pathelement location="${lib.dir}/drools-compiler-5.0.1.jar" /> <pathelement location="${lib.dir}/drools-core-5.0.1.jar" /> <pathelement location="${lib.dir}/janino-2.5.15.jar" /> </path> Here is the Java code that is throwing the error. I commented out the java.compiler code, that didn't work either. public void rules() { /* final Properties properties = new Properties(); properties.setProperty( "drools.dialect.java.compiler", "JANINO" ); PackageBuilderConfiguration cfg = new PackageBuilderConfiguration( properties ); JavaDialectConfiguration javaConf = (JavaDialectConfiguration) cfg.getDialectConfiguration( "java" ); */ final KnowledgeBuilder kbuilder = KnowledgeBuilderFactory.newKnowledgeBuilder(); // this will parse and compile in one step kbuilder.add(ResourceFactory.newClassPathResource("HelloWorld.drl", Rules.class), ResourceType.DRL); // Check the builder for errors if (kbuilder.hasErrors()) { System.out.println(kbuilder.getErrors().toString()); throw new RuntimeException("Unable to compile \"HelloWorld.drl\"."); } // Get the compiled packages (which are serializable) final Collection<KnowledgePackage> pkgs = kbuilder.getKnowledgePackages(); // Add the packages to a knowledgebase (deploy the knowledge packages). final KnowledgeBase kbase = KnowledgeBaseFactory.newKnowledgeBase(); kbase.addKnowledgePackages(pkgs); final StatefulKnowledgeSession ksession = kbase.newStatefulKnowledgeSession(); ksession.setGlobal("list", new ArrayList<Object>()); ksession.addEventListener(new DebugAgendaEventListener()); ksession.addEventListener(new DebugWorkingMemoryEventListener()); // Setup the audit logging KnowledgeRuntimeLogger logger = KnowledgeRuntimeLoggerFactory.newFileLogger(ksession, "log/helloworld"); final Message message = new Message(); message.setMessage("Hello World"); message.setStatus(Message.HELLO); ksession.insert(message); ksession.fireAllRules(); logger.close(); ksession.dispose(); } ... Here I don't think Ant is relevant because I have fork set to true: <target name="test" depends="compile"> <java classname="org.berlin.rpg.rules.Rules" fork="true"> <classpath refid="classpath.rt" /> <classpath> <pathelement location="${basedir}" /> <pathelement location="${build.classes.dir}" /> </classpath> </java> </target> The error is thrown at line 1. Basically, I haven't done anything except call final KnowledgeBuilder kbuilder = KnowledgeBuilderFactory.newKnowledgeBuilder(); I am running with Windows XP, Java6, and within Ant.1.7. The most recent (as of yesterday) version 5 of Drools-Rules.

    Read the article

  • Should this immutable struct be a mutable class?

    - by ChaosPandion
    I showed this struct to a fellow programmer and they felt that it should be a mutable class. They felt it is inconvenient not to have null references and the ability to alter the object as required. I would really like to know if there are any other reasons to make this a mutable class. [Serializable] public struct PhoneNumber : ICloneable, IEquatable<PhoneNumber> { private const int AreaCodeShift = 54; private const int CentralOfficeCodeShift = 44; private const int SubscriberNumberShift = 30; private const int CentralOfficeCodeMask = 0x000003FF; private const int SubscriberNumberMask = 0x00003FFF; private const int ExtensionMask = 0x3FFFFFFF; private readonly ulong value; public int AreaCode { get { return UnmaskAreaCode(value); } } public int CentralOfficeCode { get { return UnmaskCentralOfficeCode(value); } } public int SubscriberNumber { get { return UnmaskSubscriberNumber(value); } } public int Extension { get { return UnmaskExtension(value); } } public PhoneNumber(ulong value) : this(UnmaskAreaCode(value), UnmaskCentralOfficeCode(value), UnmaskSubscriberNumber(value), UnmaskExtension(value), true) { } public PhoneNumber(int areaCode, int centralOfficeCode, int subscriberNumber) : this(areaCode, centralOfficeCode, subscriberNumber, 0, true) { } public PhoneNumber(int areaCode, int centralOfficeCode, int subscriberNumber, int extension) : this(areaCode, centralOfficeCode, subscriberNumber, extension, true) { } private PhoneNumber(int areaCode, int centralOfficeCode, int subscriberNumber, int extension, bool throwException) { value = 0; if (areaCode < 200 || areaCode > 989) { if (!throwException) return; throw new ArgumentOutOfRangeException("areaCode", areaCode, @"The area code portion must fall between 200 and 989."); } else if (centralOfficeCode < 200 || centralOfficeCode > 999) { if (!throwException) return; throw new ArgumentOutOfRangeException("centralOfficeCode", centralOfficeCode, @"The central office code portion must fall between 200 and 999."); } else if (subscriberNumber < 0 || subscriberNumber > 9999) { if (!throwException) return; throw new ArgumentOutOfRangeException("subscriberNumber", subscriberNumber, @"The subscriber number portion must fall between 0 and 9999."); } else if (extension < 0 || extension > 1073741824) { if (!throwException) return; throw new ArgumentOutOfRangeException("extension", extension, @"The extension portion must fall between 0 and 1073741824."); } else if (areaCode.ToString()[1] - 48 > 8) { if (!throwException) return; throw new ArgumentOutOfRangeException("areaCode", areaCode, @"The second digit of the area code cannot be greater than 8."); } else { value |= ((ulong)(uint)areaCode << AreaCodeShift); value |= ((ulong)(uint)centralOfficeCode << CentralOfficeCodeShift); value |= ((ulong)(uint)subscriberNumber << SubscriberNumberShift); value |= ((ulong)(uint)extension); } } public object Clone() { return this; } public override bool Equals(object obj) { return obj != null && obj.GetType() == typeof(PhoneNumber) && Equals((PhoneNumber)obj); } public bool Equals(PhoneNumber other) { return this.value == other.value; } public override int GetHashCode() { return value.GetHashCode(); } public override string ToString() { return ToString(PhoneNumberFormat.Separated); } public string ToString(PhoneNumberFormat format) { switch (format) { case PhoneNumberFormat.Plain: return string.Format(@"{0:D3}{1:D3}{2:D4} {3:#}", AreaCode, CentralOfficeCode, SubscriberNumber, Extension).Trim(); case PhoneNumberFormat.Separated: return string.Format(@"{0:D3}-{1:D3}-{2:D4} {3:#}", AreaCode, CentralOfficeCode, SubscriberNumber, Extension).Trim(); default: throw new ArgumentOutOfRangeException("format"); } } public ulong ToUInt64() { return value; } public static PhoneNumber Parse(string value) { var result = default(PhoneNumber); if (!TryParse(value, out result)) { throw new FormatException(string.Format(@"The string ""{0}"" could not be parsed as a phone number.", value)); } return result; } public static bool TryParse(string value, out PhoneNumber result) { result = default(PhoneNumber); if (string.IsNullOrEmpty(value)) { return false; } var index = 0; var numericPieces = new char[value.Length]; foreach (var c in value) { if (char.IsNumber(c)) { numericPieces[index++] = c; } } if (index < 9) { return false; } var numericString = new string(numericPieces); var areaCode = int.Parse(numericString.Substring(0, 3)); var centralOfficeCode = int.Parse(numericString.Substring(3, 3)); var subscriberNumber = int.Parse(numericString.Substring(6, 4)); var extension = 0; if (numericString.Length > 10) { extension = int.Parse(numericString.Substring(10)); } result = new PhoneNumber( areaCode, centralOfficeCode, subscriberNumber, extension, false ); return result.value == 0; } public static bool operator ==(PhoneNumber left, PhoneNumber right) { return left.Equals(right); } public static bool operator !=(PhoneNumber left, PhoneNumber right) { return !left.Equals(right); } private static int UnmaskAreaCode(ulong value) { return (int)(value >> AreaCodeShift); } private static int UnmaskCentralOfficeCode(ulong value) { return (int)((value >> CentralOfficeCodeShift) & CentralOfficeCodeMask); } private static int UnmaskSubscriberNumber(ulong value) { return (int)((value >> SubscriberNumberShift) & SubscriberNumberMask); } private static int UnmaskExtension(ulong value) { return (int)(value & ExtensionMask); } } public enum PhoneNumberFormat { Plain, Separated }

    Read the article

  • Hibernate exception

    - by Mark
    Hi all, im new to hibernate! i have followed the netbeans tutorial on creating a hibernate enabled application. after sucessfully creating a database in mysql workbench i reversed engineered the pojos etc and then tried to run a simple query(from Course) and got the following org.hibernate.MappingException: An association from the table coursemodule refers to an unmapped class: DAL.Module at org.hibernate.cfg.Configuration.secondPassCompileForeignKeys(Configuration.java:1252) at org.hibernate.cfg.Configuration.secondPassCompile(Configuration.java:1170) at org.hibernate.cfg.AnnotationConfiguration.secondPassCompile(AnnotationConfiguration.java:324) at org.hibernate.cfg.Configuration.buildSessionFactory(Configuration.java:1286) at org.hibernate.cfg.AnnotationConfiguration.buildSessionFactory(AnnotationConfiguration.java:859) heres the generated class for Course package DAL; // Generated 02-May-2010 16:41:16 by Hibernate Tools 3.2.1.GA import java.util.HashSet; import java.util.Set; /** * Course generated by hbm2java */ public class Course implements java.io.Serializable { private int id; private String name; private Set<Module> modules = new HashSet<Module>(0); public Course() { } public Course(int id, String name) { this.id = id; this.name = name; } public Course(int id, String name, Set<Module> modules) { this.id = id; this.name = name; this.modules = modules; } public int getId() { return this.id; } public void setId(int id) { this.id = id; } public String getName() { return this.name; } public void setName(String name) { this.name = name; } public Set<Module> getModules() { return this.modules; } public void setModules(Set<Module> modules) { this.modules = modules; } } and its config file course.hbm.xml <?xml version="1.0"?> <!DOCTYPE hibernate-mapping PUBLIC "-//Hibernate/Hibernate Mapping DTD 3.0//EN" "http://hibernate.sourceforge.net/hibernate-mapping-3.0.dtd"> <!-- Generated 02-May-2010 16:41:16 by Hibernate Tools 3.2.1.GA --> <hibernate-mapping> <class name="DAL.Course" table="course" catalog="walkthrough"> <id name="id" type="int"> <column name="id" /> <generator class="assigned" /> </id> <property name="name" type="string"> <column name="name" not-null="true" /> </property> <set name="modules" inverse="false" table="coursemodule"> <key> <column name="courseId" not-null="true" unique="true" /> </key> <many-to-many entity-name="DAL.Module"> <column name="moduleId" not-null="true" unique="true" /> </many-to-many> </set> </class> </hibernate-mapping> hibernate.reveng.xml <?xml version="1.0" encoding="UTF-8"?> <!DOCTYPE hibernate-reverse-engineering PUBLIC "-//Hibernate/Hibernate Reverse Engineering DTD 3.0//EN" "http://hibernate.sourceforge.net/hibernate-reverse-engineering-3.0.dtd"> <hibernate-reverse-engineering> <schema-selection match-catalog="Walkthrough"/> <table-filter match-name="walkthrough"/> <table-filter match-name="course"/> <table-filter match-name="module"/> <table-filter match-name="studentmodule"/> <table-filter match-name="attendee"/> <table-filter match-name="student"/> <table-filter match-name="coursemodule"/> <table-filter match-name="session"/> <table-filter match-name="test"/> </hibernate-reverse-engineering> hibernate.cfg.xml <?xml version="1.0" encoding="UTF-8"?> <!DOCTYPE hibernate-configuration PUBLIC "-//Hibernate/Hibernate Configuration DTD 3.0//EN" "http://hibernate.sourceforge.net/hibernate-configuration-3.0.dtd"> <hibernate-configuration> <session-factory> <property name="hibernate.dialect">org.hibernate.dialect.MySQLDialect</property> <property name="hibernate.connection.driver_class">com.mysql.jdbc.Driver</property> <property name="hibernate.connection.url">jdbc:mysql://localhost:3306/Walkthrough</property> <property name="hibernate.connection.username">root</property> <property name="hibernate.connection.password">password</property> <property name="hibernate.show_sql">true</property> <property name="hibernate.current_session_context_class">thread</property> <mapping resource="DAL/Student.hbm.xml"/> <mapping resource="DAL/Walkthrough.hbm.xml"/> <mapping resource="DAL/Test.hbm.xml"/> <mapping resource="DAL/Module.hbm.xml"/> <mapping resource="DAL/Session.hbm.xml"/> <mapping resource="DAL/Course.hbm.xml"/> </session-factory> </hibernate-configuration> any ideas on why im getting this exception? ps. test is just a table with an id in it and is not related to anything. running "from Test" works

    Read the article

  • CodePlex Daily Summary for Friday, June 03, 2011

    CodePlex Daily Summary for Friday, June 03, 2011Popular ReleasesMedia Companion: MC 3.404b Weekly: Extract the entire archive to a folder which has user access rights, eg desktop, documents etc. Refer to the documentation on this site for the Installation & Setup Guide Important! *** Due to an issue where the date added & the full genre information was not being read into the Movie cache, it is highly recommended that you perform a Rebuild Movies when first running this latest version. This will read in the information from the nfo's & repopulate the cache used by MC during operation. Fi...Terraria Map Generator: TerrariaMapTool 1.0.0.4 Beta: 1) Fixed the generated map.html file so that the file:/// is included in the base path. 2) Added the ability to use parallelization during generation. This will cause the program to use as many threads as there are physical cores. 3) Fixed some background overdraw.DotRas: DotRas v1.2 (Version 1.2.4168): This release includes compiled (and signed) versions of the binaries, PDBs, CHM help documentation, along with both C# and VB.NET examples. Please don't forget to rate the release! If you find a bug, please open a work item and give as much description as possible. Stack traces, which operating system(s) you're targeting, and build type is also very helpful for bug hunting. If you find something you believe to be a bug but are not sure, create a new discussion on the discussions board. Thank...Ajax Minifier: Microsoft Ajax Minifier 4.21: Fixed issues #15949 and #15868. Tweaked output in multi-line (pretty-print) mode so that var-statements in the initializer of for-statements break multiple variables onto multiple lines. Added TreeModification flag (and can now therefore turn off) the behavior of removing quotes around object literal property names if the names can be valid identifiers. Convert true literals to !0 and false literals to !1. Added a Visitor class approach to iterating over a generated abstract syntax tree to he...Caliburn Micro: WPF, Silverlight and WP7 made easy.: Caliburn.Micro v1.1 RTW: Download ContentsDebug and Release Assemblies Samples Changes.txt License.txt Release Highlights For WP7A new Tombstoning API based on ideas from Fluent NHibernate A new Launcher/Chooser API Strongly typed Navigation SimpleContainer included The full phone lifecycle is made easy to work with ie. we figure out whether your app is actually Resurrecting or just Continuing for you For WPFSupport for the Client Profile Better support for WinForms integration All PlatformsA power...VidCoder: 0.9.1: Added color coding to the Log window. Errors are highlighted in red, HandBrake logs are in black and VidCoder logs are in dark blue. Moved enqueue button to the right with the other control buttons. Added logic to report failures when errors are logged during the encode or when the encode finishes prematurely. Added Copy button to Log window. Adjusted audio track selection box to always show the full track name. Changed encode job progress bar to also be colored yellow when the enco...AutoLoL: AutoLoL v2.0.1: - Fixed a small bug in Auto Login - Fixed the updaterEPPlus-Create advanced Excel 2007 spreadsheets on the server: EPPlus 2.9.0.1: EPPlus-Create advanced Excel 2007 spreadsheets on the server This version has been updated to .Net Framework 3.5 New Features Data Validation. PivotTables (Basic functionalliy...) Support for worksheet data sources. Column, Row, Page and Data fields. Date and Numeric grouping Build in styles. ...and more And some minor new features... Ranges Text-Property|Get the formated value AutofitColumns-method to set the column width from the content of the range LoadFromCollection-metho...jQuery ASP.Net MVC Controls: Version 1.4.0.0: Version 1.4.0.0 contains the following additions: Upgraded to MVC 3.0 Upgraded to jQuery 1.6.1 (Though the project supports all jQuery version from 1.4.x onwards) Upgraded to jqGrid 3.8 Better Razor View-Engine support Better Pager support, includes support for custom pagers Added jqGrid toolbar buttons support Search module refactored, with full suport for multiple filters and ordering And Code cleanup, bug-fixes and better controller configuration support.Nearforums - ASP.NET MVC forum engine: Nearforums v6.0: Version 6.0 of Nearforums, the ASP.NET MVC Forum Engine, containing new features: Authentication using Membership Provider for SQL Server and MySql Spam prevention: Flood Control Moderation: Flag messages Content management: Pages: Create pages (about us/contact/texts) through web administration Allow nearforums to run as an IIS subapp Migrated Facebook Connect to OAuth 2.0 Visit the project Roadmap for more details.NetOffice - The easiest way to use Office in .NET: NetOffice Release 0.8b: Changes: - fix critical issue 15922(AccessViolationException) once and for all ...update is strongly recommended Known Issues: - some addin ribbon examples has a COM Register function with missing codebase entry(win32 registry) ...the problem is only affected to c# examples. fixed in latest source code. NetOffice\ReleaseTags\NetOffice Release 0.8.rar Includes: - Runtime Binaries and Source Code for .NET Framework:......v2.0, v3.0, v3.5, v4.0 - Tutorials in C# and VB.Net:...................Facebook Graph Toolkit: Facebook Graph Toolkit 1.5.4186: Updates the API in response to Facebook's recent change of policy: All Graph Api accessing feeds or posts must provide a AccessToken.Serviio for Windows Home Server: Beta Release 0.5.2.0: Ready for widespread beta. Synchronized build number to Serviio version to avoid confusion.AcDown????? - Anime&Comic Downloader: AcDown????? v3.0 Beta4: ??AcDown?????????????,??????????????,????、????。?????Acfun????? ????32??64? Windows XP/Vista/7 ????????????? ??:????????Windows XP???,?????????.NET Framework 2.0???(x86)?.NET Framework 2.0???(x64),?????"?????????"??? ??v3.0 Beta4 2011-5-31?? ???Bilibili.us????? ???? ?? ???"????" ???Bilibili.us??? ??????? ?? ??????? ?? ???????? ?? ?? ???Bilibili.us?????(??????????????????) ??????(6.cn)?????(????) ?? ?????Acfun?????????? ?????????????? ???QQ???????? ????????????Discussion...CodeCopy Auto Code Converter: Code Copy v0.1: Full add-in, setup project source code and setup fileWordpress Theme 'Windows Metro': v1.00.0017: v1.00.0017 Dies ist eine Vorab-Version aus der Entwickler-Phase. This is a preview version from the development phase.TerrariViewer: TerrariViewer v2.4.1: Added Piggy Bank editor and fixed some minor bugs.Kooboo CMS: Kooboo CMS 3.02: Updated the Kooboo_CMS.zip at 2011-06-02 11:44 GMT 1.Fixed: Adding data rule issue on page. 2.Fixed: Caching issue for higher performance. Updated the Kooboo_CMS.zip at 2011-06-01 10:00 GMT 1. Fixed the published package throwed a compile error under WebSite mode. 2. Fixed the ContentHelper.NewMediaFolderObject return TextFolder issue. 3. Shorten the name of ContentHelper API. NewMediaFolderObject=>MediaFolder, NewTextFolderObject=> TextFolder, NewSchemaObject=>Schema. Also update the C...mojoPortal: 2.3.6.6: see release notes on mojoportal.com http://www.mojoportal.com/mojoportal-2366-released Note that we have separate deployment packages for .NET 3.5 and .NET 4.0 The deployment package downloads on this page are pre-compiled and ready for production deployment, they contain no C# source code. To download the source code see the Source Code Tab I recommend getting the latest source code using TortoiseHG, you can get the source code corresponding to this release here.Terraria World Creator: Terraria World Creator: Version 1.01 Fixed a bug that would cause the application to crash. Re-named the Application.New Projects"Background Transfer Service Sample" From WP7 Sample: This project is an extension to existing Windows Phone7 sample available from MS sight ("Background Transfer Service Sample"). This project extends functionality to display the background downloaded item to verify download conten. Agile project and development management dashboard - Meter Console: Agile project and development management dashboard application. The focus of this application to bridge the gap between the management, clients and developer and chive best possible transparency. Ajax Multi-ListBox: Ajax Multi-ListBox is .NET C# written user control with a dual listbox that uses ajax to move items from left to right and vice versa.aLOG : Asynchronous logging (Logging Library): aLog is an asynchronous logging component built on Microsoft Enterprise Exception handling and Logging Component block. It allows application to asynchronously log the data in multiple data sources such as file system, database , XML. Application Base Component Library: Application Base Component LibraryConcise Service Bus: A compact but comprehensive WCF service bus framework.Contour provider for Umbraco Courier: Transfer Umbraco Contour forms between Umbraco instances with Courier.CoveyNet MidWest: CoveyNet MidWestCSharp Samples: This contain C#- wpf, wcf, linqtosql, some basic oops stuff etc.DotNet1: DotNet1FireWeb: fefaesfrafasGeoCoder: Basic server side GeoCoder which accesses Google, YAHOO etc. This is developed in C# and is a mix of language styles, just depended what I was doing that week. All the libraries include tests which can be used to check if the libraries still work, it's worth doing this as the consumed webservices may change from time to time. GrandReward: GrandReward is designed to help young and old people to learn by answer questions. It is easy to use and the community can design own topic files for own questions to learn. The project is develop in C# and contains some WPF views. The project homepage is: http://grand-reward.com/helferlein: The helferlein library contains some web controls and tools I used in other projects.helferlein_BabelFish: helferlein_BabelFish is a DNN module to support multilingual features for other helferlein modules. It is developed in C#.helferlein_Form: helferlein_Form is an easy-to-use module for DotNetNuke that allows creating forms. The submissions can be sent by e-mail and/or stored in a database. It's developed in C#.ILSpy/Silverlight — Proof Of Concept: ILSpy refactoring (dire hacking) to make it work in Silverlight.ISGProject: ISGProjectMarcelo Rocha: Trabalhos acadêmicos, Apresentações, Projetos e outras Invencionices de Marcelo Rocha.MosquitoOnline: MosquitoOnline ProjectNET.Studio: Ein kleines Visual Studio für die SchuleOmegle Desktop Client: A desktop client for the anonymous chat site, Omegle. Designed to combat some of the shortcomings of the webpage, the desktop client will have no adverts, and be able to flash the taskbar window, and play a sound on message received. Online PHP IDE: Free Open Source online editor for working with files on FTP. Free and easy deployment.Open JGL: A utillity library using OpenGL, which focus' on graphics and vertex control.Open Tokenization Server: Open Tokenization Server is a simplistic implementation of a tokenization data security service. It allows users to create and retrieve tokens that represent sensitive information.OurCodeNights: Just some small projects we "work" on at our codenights. A good excuse to meet and have a good time ;)Raple: Raple is simple programming languageSea: Set of C# Extension libraries that make life easier.Shai Chi: Shai ChiSharePoint Sandbox Logging: SharePoint Sandbox Logging enables developers and solution designers to easily log events to logging list in your site collection, allowing you to easily trace errors on custom solutions within your site collection, with no need for server access. Perfect for sandbox / Office365. This project contains one sandbox solution with a site collection logging feature. When feature is active - it creates the list and logs. If it is not active - it does not log. Simple. It also contains a sampl...SharePoint Stock Ticker: SharePoint Stock Ticker is a SharePoint 2010 project which consists of a SharePoint Stock Ticker Web Part driven off of Google's Stock API. The width of this Web Part was developed to fit within the default SharePoint 2010 Quick Launch navigation. This has also been tested to run over http and https SharePoint 2010 farms and will not throw a mixed content warning with Internet Explorer.SharpLinx: SharpLinx is an open source social networking application on ASP.Net.Single.Net - less Copy & Paste,less code lines,more easier: make your code life easierSqlMyAdmin: A projecto to create a PHPMyAdmin clone app that access Sql Server.TDB: TDB is a NOSQL Key-value store entirely designed to run in windows environment. It's directly inspired by Redis project and use Redis' command sematincs and Redis' communication protocol to maintain a base compatibility with it.Tomasulo: A simulator of Tomasulo CDB dynamic instruction scheduling algorithm in Java. Authors: Mengyu Zhou @J85 Zhenyao Zhu @J85 Jun Li @J81 @Tsinghua UniversityTransport Broker Management System: Transport Management system will serve the small freight brokers manage the carriers and orders. It has features like manage truck owners, carriers, source, destination, orders. Verification system for MCBS: Verification system for mobile communication base station.VietCard: VietCardVkontakte File converter: ????????? ?????? ??? ???????? ?? ? "?????????" vkontakte.ru. VSGesture for Visual Studio: VSGesture can execute command via mouse gestures within Visual Studio2010. If you have any feedback, please send me an email to powerumc at gmail.com.WCF-Silverlight Helper: Class library to generate WCF DataContractSerializer-serializable and Silverlight (SL) -compatible classes. Useful, for example, if you have an assembly with classes that fail to serialize with DataContractSerializer. T4 Templates are used to auto-generate code.Windows Phone eBook Samples: Windows Phone eBook Samples is a collection of samples associated with the Windows Phone eBoom community initiative.WinPreciousMetal: WinPreciousMetal helps people know the precious metal's states. Because I am a Chinese, my soft mainly focus at the price of metal in RMB.

    Read the article

  • CodePlex Daily Summary for Saturday, November 12, 2011

    CodePlex Daily Summary for Saturday, November 12, 2011Popular ReleasesDynamic PagedCollection (Silverlight / WPF Pagination): PagedCollection: All classes which facilitate your dynamic pagination in Silverlight or WPF !Media Companion: MC 3.422b Weekly: Ensure .NET 4.0 Full Framework is installed. (Available from http://www.microsoft.com/download/en/details.aspx?id=17718) Ensure the NFO ID fix is applied when transitioning from versions prior to 3.416b. (Details here) TV Show Resolutions... Made the TV Shows folder list sorted. Re-visibled 'Manually Add Path' in Root Folders. Sorted list to process during new tv episode search Rebuild Movies now processes thru folders alphabetically Fix for issue #208 - Display Missing Episodes is not popu...DotSpatial: DotSpatial Release Candidate 1 (1.0.823): Supports loading extensions using System.ComponentModel.Composition. DemoMap compiled as x86 so that GDAL runs on x64 machines. How to: Use an Assembly from the WebBe aware that your browser may add an identifier to downloaded files which results in "blocked" dll files. You can follow the following link to learn how to "Unblock" files. Right click on the zip file before unzipping, choose properties, go to the general tab and click the unblock button. http://msdn.microsoft.com/en-us/library...XPath Visualizer: XPathVisualizer v1.3 Latest: This is v1.3.0.6 of XpathVisualizer. This is an update release for v1.3. These workitems have been fixed since v1.3.0.5: 7429 7432 7427MSBuild Extension Pack: November 2011: Release Blog Post The MSBuild Extension Pack November 2011 release provides a collection of over 415 MSBuild tasks. A high level summary of what the tasks currently cover includes the following: System Items: Active Directory, Certificates, COM+, Console, Date and Time, Drives, Environment Variables, Event Logs, Files and Folders, FTP, GAC, Network, Performance Counters, Registry, Services, Sound Code: Assemblies, AsyncExec, CAB Files, Code Signing, DynamicExecute, File Detokenisation, GU...Quick Performance Monitor: Version 1.8.0: Now you can add multiple counters/instances at once! Yes, I know its overdue...CODE Framework: 4.0.11110.0: Various minor fixes and tweaks.Extensions for Reactive Extensions (Rxx): Rxx 1.2: What's NewRelated Work Items Please read the latest release notes for details about what's new. Content SummaryRxx provides the following features. See the Documentation for details. Many IObservable<T> extension methods and IEnumerable<T> extension methods. Many useful types such as ViewModel, CommandSubject, ListSubject, DictionarySubject, ObservableDynamicObject, Either<TLeft, TRight>, Maybe<T> and others. Various interactive labs that illustrate the runtime behavior of the extensio...Player Framework by Microsoft: HTML5 Player Framework 1.0: Additional DownloadsHTML5 Player Framework Examples - This is a set of examples showing how to setup and initialize the HTML5 Player Framework. This includes examples of how to use the Player Framework with both the HTML5 video tag and Silverlight player. Note: Be sure to unblock the zip file before using. Note: In order to test Silverlight fallback in the included sample app, you need to run the html and xap files over http (e.g. over localhost). Silverlight Players - Visit the Silverlig...MapWindow 4: MapWindow GIS v4.8.6 - Final release - 64Bit: What’s New in 4.8.6 (Final release)A few minor issues have been fixed What’s New in 4.8.5 (Beta release)Assign projection tool. (Sergei Leschinsky) Projection dialects. (Sergei Leschinsky) Projections database converted to SQLite format. (Sergei Leschinsky) Basic code for database support - will be developed further (ShapefileDataClient class, IDataProvider interface). (Sergei Leschinsky) 'Export shapefile to database' tool. (Sergei Leschinsky) Made the GEOS library static. geos.dl...NewLife XCode ??????: XCode v8.2.2011.1107、XCoder v4.5.2011.1108: v8.2.2011.1107 ?IEntityOperate.Create?Entity.CreateInstance??????forEdit,????????(FindByKeyForEdit)???,???false ??????Entity.CreateInstance,????forEdit,???????????????????? v8.2.2011.1103 ??MS????,??MaxMin??(????????)、NotIn??(????)、?Top??(??NotIn)、RowNumber??(?????) v8.2.2011.1101 SqlServer?????????DataPath,?????????????????????? Oracle?????????DllPath,????OCI??,???????????ORACLE_HOME?? Oracle?????XCode.Oracle.IsUseOwner,???????????Ow...Facebook C# SDK: v5.3.2: This is a RTW release which adds new features and bug fixes to v5.2.1. Query/QueryAsync methods uses graph api instead of legacy rest api. removed dependency from Code Contracts enabled Task Parallel Support in .NET 4.0+ (experimental) added support for early preview for .NET 4.5 (binaries not distributed in codeplex nor nuget.org, will need to manually build from Facebook-Net45.sln) added additional method overloads for .NET 4.5 to support IProgress<T> for upload progress added ne...Delete Inactive TS Ports: List and delete the Inactive TS Ports: UPDATEAdded support for windows 2003 servers and removed some null reference errors when the registry key was not present List and delete the Inactive TS Ports - The InactiveTSPortList.EXE accepts command line arguments The InactiveTSPortList.Standalone.WithoutPrompt.exe runs as a standalone exe without the need for any command line arguments.Ribbon Editor for Microsoft Dynamics CRM 2011: Ribbon Editor (0.1.2207.267): BUG FIXES: - Cannot add multiple JavaScript and Url under Actions - Cannot add <Or> node under <OrGroup> - Adding a rule under <Or> node put the new rule node at the wrong placeWF Designer Express: WF Designer Express v0.6: ?????(???????????) Multi Language(Now, Japanese and English)ClosedXML - The easy way to OpenXML: ClosedXML 0.60.0: Added almost full support for auto filters (missing custom date filters). See examples Filter Values, Custom Filters Fixed issues 7016, 7391, 7388, 7389, 7198, 7196, 7194, 7186, 7067, 7115, 7144Microsoft Research Boogie: Nightly builds: This download category contains automatically released nightly builds, reflecting the current state of Boogie's development. We try to make sure each nightly build passes the test suite. If you suspect that was not the case, please try the previous nightly build to see if that really is the problem. Also, please see the installation instructions.GoogleMap Control: GoogleMap Control 6.0: Major design changes to the control in order to achieve better scalability and extensibility for the new features comming with GoogleMaps API. GoogleMap control switched to GoogleMaps API v3 and .NET 4.0. GoogleMap control is 100% ScriptControl now, it requires ScriptManager to be registered on the pages where and before it is used. Markers, polylines, polygons and directions were implemented as ExtenderControl, instead of being inner properties of GoogleMap control. Better perfomance. Better...WDTVHubGen - Adds Metadata, thumbnails and subtitles to WDTV Live Hubs: V2.1: Version 2.1 (click on the right) this uses V4.0 of .net Version 2.1 adds the following features: (apologize if I forget some, added a lot of little things) Manual Lookup with TV or Movie (finally huh!), you can look up a movie or TV episode directly, you can right click on anythign, and choose manual lookup, then will allow you to type anything you want to look up and it will assign it to the file you right clicked. No Rename: a very popular request, this is an option you can set so that t...SubExtractor: Release 1020: Feature: added "baseline double quotes" character to selector box Feature: added option to save SRT files as ANSI (instead of previous UTF-8 only) Feature: made "Save Sup files to Source directory" apply to both Sup and Idx source files. Fix: removed SDH text (...) or [...] that is split over 2 lines Fix: better decision-making in when to prefix a line with a '-' because SDH was removedNew Projects- Gemini - code branch: the branch for all Hzj_jie's code(MPLF) MediaPortal LoveFilm Plugin: MediaPortal LoveFilm (Love Film) Plugin. Browse films, add/remove from lists, streams etc._: _Batalha Naval em C#: Jogo de batalha naval para a disciplina de C#BusyHoliday - Making Outlook's Calendar Busy for Holidays: When users add holidays to their Outlook calendar these are added as an all day event. However, these are added as free time. With users in other countries, if they were to invite a user in a country with a public holiday, they won't be able to see that the user is on a public holiday or busy. BusyHoliday will update existing holidays as busy time. This may improve users ability to collaborate efficiently across geographical boundries. C# and MCU: C# and MCU and webCampo Minado C Sharp: C sharp source code of the game minefield.CP2011test: CP2011 TestDynamic PagedCollection (Silverlight / WPF Pagination): PagedCollection helps a developer to easily control the pagination of their collection. This collection doesnt need all datas in memory, it can be attached to RIA/SQL/XML data ... and load needed data one after the other. The PagedCollection can be used with Silverlight or WPF.Fast MVVM: The faster way to build your MVVM (Model, View, ViewModel) applications. It's a basic and educational way to learn and work with MVVM. We have more than just a pack of classes or templates. We have a new concept that turn it a little easier.Habitat: Core game services, frameworks and templates for desktop, mobile and web game developers.Historical Data Repository: Repository stores serializable timestamped data items in compressed files structured in a way that allows to quickly find data by time, quickly read and write data items in chronological order. Key features: - all data is stored under a single root folder; like file system, repository can contain a tree of folders each of which contains its own data stream; when reading, multiple streams can be combined into a single output stream maintaining chronological order; the tree structure can b...Immunity Buster: This game is for HIV. We will be making a virus which will help to spread the knowledge about the virus and have Fun among the peers.JoeCo Blog Application: JoeCo Blog Application for ECE 574jweis sso user center: jweis is a user center,distination is to use in a contry ,ssoKinect Shape Game Connect with Windows Phone: The classic Shape Game for Microsoft Kinect extended to allow players to join in the game and deploy shapes from their windows phone. Uses TCP sockets. For educational purposes. Free to download and distribute.Liekhus Behavior Driven DevExpress XAF: Using the BDD (Behavior Driven Development) techniques of SpecFlow and the EasyTest scriptability of DevExpress XAF (eXpress Application Framework), you can leverage test first style of application development in a speed never though imaginable.Linq to BBYOpen: C# Linq Provider to access the REST based BBYOpen services from BestBuy.Mi clima: This is the source code for the application "Mi clima" published in the Windows Phone Marketplace. MySqlQueryToolkit: This project bundles all of the common performance tests into a single handy user interface. Simply drop your query in and use the tools to make your query faster. Includes index suggestion, column data type and length suggestion and common profiling options.OData Validation Framework: Project to show how to validate data using client and server side OData/WCF Data ServicesOMR Table Data Syncronizer - Only Data: This tools compare and syncronize table data. No schema syncronizing included on this project. Data comparition is fastest! 1,000,000 data compare time is: ~200 ms.OrchardPo: This is the po file management module that is used on http://orchardproject.netResistance Calculator: A calculator for parallel and/or series resistanceSMS Team 5: h? th?ng g?i SMS t? d?ngSportsStore: Prueba en tfs de sports storeUDP Transport: This is an attempt at implementation of WCF UDP transport compliant with soa.udp OASIS Standard WebUtils: Biblioteca com funções genéricas úteis para uso com ASP NET MVC??a??e?a - Greek Government Open Data Program - Windows Mobile Client: ? efa?µ??? Open Government sa? d??e? ?µes? p??sßas? ap? t? ????t? sa? se ????? t??? ??µ???, ??e? t?? ap?f?se?? ?a? ??e? t?? dap??e? t?? ????????? ???t??? ?a? ???? t?? ?p??es??? t?? ????????? ??µ?s??? µ?s? t?? ??ße???t???? p?????µµat?? ??a??e?a.

    Read the article

  • Adding a JPanel to another JPanel having TableLayout

    - by user253530
    I am trying to develop a map editor in java. My map window receives as a constructor a Map object. From that map object i am able to retrieve the Grid and every item in the grid along with other getters and setters. The problem is that even though the Mapping extends JComponent, when I place it in a panel it is not painted. I have overridden the paint method to satisfy my needs. Here is the code, maybe you could help me. public class MapTest extends JFrame implements ActionListener { private JPanel mainPanel; private JPanel mapPanel; private JPanel minimapPanel; private JPanel relationPanel; private TableLayout tableLayout; private JPanel tile; MapTest(Map map) { mainPanel = (JPanel) getContentPane(); mapPanel = new JPanel(); populateMapPanel(map); mainPanel.add(mapPanel); this.setPreferredSize(new Dimension(800, 600)); this.setDefaultCloseOperation(JFrame.EXIT_ON_CLOSE); this.setVisible(true); } private double[][] generateTableLayoutSize(int x, int y, int size) { double panelSize[][] = new double[x][y]; for (int i = 0; i < x; i++) { for (int j = 0; j < y; j++) { panelSize[i][j] = size; } } return panelSize; } private void populateMapPanel(Map map) { double[][] layoutSize = generateTableLayoutSize(map.getMapGrid().getRows(), map.getMapGrid().getColumns(), 50); tableLayout = new TableLayout(layoutSize); for(int i = 0; i < map.getMapGrid().getRows(); i++) { for(int j = 0; j < map.getMapGrid().getColumns(); j++) { tile = new JPanel(); tile.setName(String.valueOf(((Mapping)map.getMapGrid().getItem(i, j)).getCharacter())); tile.add(map.getMapItem(i, j)); String constraint = i + "," + j; mapPanel.add(tile, constraint); } } mapPanel.validate(); mapPanel.repaint(); } public void actionPerformed(ActionEvent e) { throw new UnsupportedOperationException("Not supported yet."); } } My Mapping Class public class Mapping extends JComponent implements Serializable{ private BufferedImage image; private Character character; //default public Mapping() { super(); this.image = null; this.character = '\u0000'; } //Mapping from image and char public Mapping(BufferedImage image, char character) { super(); this.image = image; this.character = character; } //Mapping from file and char public Mapping(File file, char character) { try { this.image = ImageIO.read(file); this.character = character; } catch (IOException ex) { System.out.println(ex); } } public char getCharacter() { return character; } public void setCharacter(char character) { this.character = character; } public BufferedImage getImage() { return image; } public void setImage(BufferedImage image) { this.image = image; repaint(); } @Override /*Two mappings are consider the same if -they have the same image OR -they have the same character OR -both of the above*/ public boolean equals(Object mapping) { if (this == mapping) { return true; } if (mapping instanceof Mapping) { return true; } //WARNING! equals might not work for images return (this.getImage()).equals(((Mapping) mapping).getImage()) || (this.getCharacter()) == (((Mapping) mapping).getCharacter()); } @Override public void paintComponent(Graphics g) { super.paintComponent(g); //g.drawImage(image, 0, 0, null); g.drawImage(image, 0, 0, this.getWidth(), this.getHeight(), null); } // @Override // public Dimension getPreferredSize() { // if (image == null) { // return new Dimension(10, 10); //instead of 100,100 set any prefered dimentions // } else { // return new Dimension(100, 100);//(image.getWidth(null), image.getHeight(null)); // } // } private void readObject(java.io.ObjectInputStream in) throws IOException, ClassNotFoundException { character = (Character) in.readObject(); image = ImageIO.read(ImageIO.createImageInputStream(in)); } private void writeObject(java.io.ObjectOutputStream out) throws IOException { out.writeObject(character); ImageWriter writer = (ImageWriter) ImageIO.getImageWritersBySuffix("jpg").next(); writer.setOutput(ImageIO.createImageOutputStream(out)); ImageWriteParam param = writer.getDefaultWriteParam(); param.setCompressionMode(ImageWriteParam.MODE_EXPLICIT); param.setCompressionQuality(0.85f); writer.write(null, new IIOImage(image, null, null), param); } }

    Read the article

  • testing dao with hibernate genericdao pattern with spring.Headache

    - by black sensei
    Hello good fellas! in my journey of learning hibernate i came across an article on hibernate site. i' learning spring too and wanted to do certain things to discover the flexibility of spring by letting you implement you own session.yes i don't want to use the hibernateTemplate(for experiment). and i'm now having a problem and even the test class.I followed the article on the hibernate site especially the section an "implementation with hibernate" so we have the generic dao interface : public interface GenericDAO<T, ID extends Serializable> { T findById(ID id, boolean lock); List<T> findAll(); List<T> findByExample(T exampleInstance); T makePersistent(T entity); void makeTransient(T entity); } it's implementation in an abstract class that is the same as the one on the web site.Please refer to it from the link i provide.i'll like to save this post to be too long now come my dao's messagedao interface package com.project.core.dao; import com.project.core.model.MessageDetails; import java.util.List; public interface MessageDAO extends GenericDAO<MessageDetails, Long>{ //Message class is on of my pojo public List<Message> GetAllByStatus(String status); } its implementation is messagedaoimpl: public class MessageDAOImpl extends GenericDAOImpl <Message, Long> implements MessageDAO { // mySContainer is an interface which my HibernateUtils implement mySContainer sessionManager; /** * */ public MessageDAOImpl(){} /** * * @param sessionManager */ public MessageDAOImpl(HibernateUtils sessionManager){ this.sessionManager = sessionManager; } //........ plus other methods } here is my HibernatUtils public class HibernateUtils implements SessionContainer { private final SessionFactory sessionFactory; private Session session; public HibernateUtils() { this.sessionFactory = new AnnotationConfiguration().configure().buildSessionFactory(); } public HibernateUtils(SessionFactory sessionFactory) { this.sessionFactory = sessionFactory; } /** * * this is the function that return a session.So i'm free to implements any type of session in here. */ public Session requestSession() { // if (session != null || session.isOpen()) { // return session; // } else { session = sessionFactory.openSession(); // } return session; } } So in my understanding while using spring(will provide the conf), i'ld wire sessionFactory to my HiberbernateUtils and then wire its method RequestSession to the Session Property of the GenericDAOImpl (the one from the link provided). here is my spring config core.xml <bean id="sessionManager" class="com.project.core.dao.hibernate.HibernateUtils"> <constructor-arg ref="sessionFactory" /> </bean> <bean id="messageDao" class="com.project.core.dao.hibernate.MessageDAOImpl"> <constructor-arg ref="sessionManager"/> </bean> <bean id="genericDAOimpl" class="com.project.core.dao.GenericDAO"> <property name="session" ref="mySession"/> </bean> <bean id="mySession" factory-bean="com.project.core.dao.SessionContainer" factory-method="requestSession"/> now my test is this public class MessageDetailsDAOImplTest extends AbstractDependencyInjectionSpringContextTests{ HibernateUtils sessionManager = (HibernateUtils) applicationContext.getBean("sessionManager"); MessageDAO messagedao =(MessageDAO) applicationContext.getBean("messageDao"); static Message[] message = new Message[] { new Message("text",1,"test for dummies 1","1234567890","Pending",new Date()), new Message("text",2,"test for dummies 2","334455669990","Delivered",new Date()) }; public MessageDAOImplTest() { } @Override protected String[] getConfigLocations(){ return new String[]{"file:src/main/resources/core.xml"}; } @Test public void testMakePersistent() { System.out.println("MakePersistent"); messagedao.makePersistent(message[0]); Session session = sessionManager.RequestSession(); session.beginTransaction(); MessageDetails fromdb = ( Message) session.load(Message.class, message[0].getMessageId()); assertEquals(fromdb.getMessageId(), message[0].getMessageId()); assertEquals(fromdb.getDateSent(),message.getDateSent()); assertEquals(fromdb.getGlobalStatus(),message.getGlobalStatus()); assertEquals(fromdb.getNumberOfPages(),message.getNumberOfPages()); } i'm having this error exception in constructor testMakePersistent(java.lang.NullPointerException at com.project.core.dao.hibernate.MessageDAOImplTest) with this stack : at com.project.core.dao.hibernate.MessageDAOImplTest.(MessageDAOImplTest.java:28) at sun.reflect.NativeConstructorAccessorImpl.newInstance0(Native Method) at sun.reflect.NativeConstructorAccessorImpl.newInstance(NativeConstructorAccessorImpl.java:39) at sun.reflect.DelegatingConstructorAccessorImpl.newInstance(DelegatingConstructorAccessorImpl.java:27) at java.lang.reflect.Constructor.newInstance(Constructor.java:513) at junit.framework.TestSuite.createTest(TestSuite.java:61) at junit.framework.TestSuite.addTestMethod(TestSuite.java:283) at junit.framework.TestSuite.(TestSuite.java:146) at org.apache.tools.ant.taskdefs.optional.junit.JUnitTestRunner.run(JUnitTestRunner.java:481) at org.apache.tools.ant.taskdefs.optional.junit.JUnitTestRunner.launch(JUnitTestRunner.java:1031) at org.apache.tools.ant.taskdefs.optional.junit.JUnitTestRunner.main(JUnitTestRunner.java:888) )) How to actually make this one work.I know this is a lot to stuffs and i'm thanking you for reading it.Please give me a solution.How would you do this? thanks

    Read the article

  • Hibernate Query Language Problem

    - by Sarang
    Well, I have implemented a distinct query in hibernate. It returns me result. But, while casting the fields are getting interchanged. So, it generates casting error. What should be the solution? As an example, I do have database, "ProjectAssignment" that has three fields, aid, pid & userName. I want all distinct userName data from this table. I have applied query : select distinct userName, aid, pid from ProjectAssignment Whereas the ProjectAssignment.java file has the fields in sequence aid, pid & userName. Now, here the userName is first field in output. So, Casting is not getting possible. Also, query : select aid, pid, distinct userName from ProjectAssignment is not working. What is the proper query for the same ? Or what else the solution ? The code is as below : System Utilization Service Bean Method where I have to retrieve data : public List<ProjectAssignment> getProjectAssignments() { projectAssignments = ProjectAssignmentHelper.getAllResources(); //Here comes the error return projectAssignments; } ProjectAssignmentHelper from where I fetch Data : package com.hibernate; import java.util.List; import org.hibernate.Query; import org.hibernate.Session; public class ProjectAssignmentHelper { public static List<ProjectAssignment> getAllResources() { List<ProjectAssignment> projectMasters; Session session = HibernateUtil.getSessionFactory().openSession(); Query query = session.createQuery("select distinct aid, pid, userName from ProjectAssignment"); projectMasters = (List<ProjectAssignment>) query.list(); session.close(); return projectMasters; } } Hibernate Data Bean : package com.hibernate; public class ProjectAssignment implements java.io.Serializable { private short aid; private String pid; private String userName; public ProjectAssignment() { } public ProjectAssignment(short aid) { this.aid = aid; } public ProjectAssignment(short aid, String pid, String userName) { this.aid = aid; this.pid = pid; this.userName = userName; } public short getAid() { return this.aid; } public void setAid(short aid) { this.aid = aid; } public String getPid() { return this.pid; } public void setPid(String pid) { this.pid = pid; } public String getUserName() { return this.userName; } public void setUserName(String userName) { this.userName = userName; } } Error : For input string: "userName" java.lang.NumberFormatException: For input string: "userName" at java.lang.NumberFormatException.forInputString(NumberFormatException.java:48) at java.lang.Integer.parseInt(Integer.java:447) at java.lang.Integer.parseInt(Integer.java:497) at javax.el.ArrayELResolver.toInteger(ArrayELResolver.java:375) at javax.el.ArrayELResolver.getValue(ArrayELResolver.java:195) at javax.el.CompositeELResolver.getValue(CompositeELResolver.java:175) at com.sun.faces.el.FacesCompositeELResolver.getValue(FacesCompositeELResolver.java:72) at com.sun.el.parser.AstValue.getValue(AstValue.java:116) at com.sun.el.parser.AstValue.getValue(AstValue.java:163) at com.sun.el.ValueExpressionImpl.getValue(ValueExpressionImpl.java:219) at com.sun.faces.facelets.el.TagValueExpression.getValue(TagValueExpression.java:102) at javax.faces.component.ComponentStateHelper.eval(ComponentStateHelper.java:190) at javax.faces.component.ComponentStateHelper.eval(ComponentStateHelper.java:178) at javax.faces.component.UICommand.getValue(UICommand.java:218) at org.primefaces.component.commandlink.CommandLinkRenderer.encodeMarkup(CommandLinkRenderer.java:113) at org.primefaces.component.commandlink.CommandLinkRenderer.encodeEnd(CommandLinkRenderer.java:54) at javax.faces.component.UIComponentBase.encodeEnd(UIComponentBase.java:878) at org.primefaces.renderkit.CoreRenderer.renderChild(CoreRenderer.java:70) at org.primefaces.renderkit.CoreRenderer.renderChildren(CoreRenderer.java:54) at org.primefaces.component.datatable.DataTableRenderer.encodeTable(DataTableRenderer.java:525) at org.primefaces.component.datatable.DataTableRenderer.encodeMarkup(DataTableRenderer.java:407) at org.primefaces.component.datatable.DataTableRenderer.encodeEnd(DataTableRenderer.java:193) at javax.faces.component.UIComponentBase.encodeEnd(UIComponentBase.java:878) at org.primefaces.renderkit.CoreRenderer.renderChild(CoreRenderer.java:70) at org.primefaces.renderkit.CoreRenderer.renderChildren(CoreRenderer.java:54) at org.primefaces.component.tabview.TabViewRenderer.encodeContents(TabViewRenderer.java:198) at org.primefaces.component.tabview.TabViewRenderer.encodeMarkup(TabViewRenderer.java:130) at org.primefaces.component.tabview.TabViewRenderer.encodeEnd(TabViewRenderer.java:48) at javax.faces.component.UIComponentBase.encodeEnd(UIComponentBase.java:878) at javax.faces.component.UIComponent.encodeAll(UIComponent.java:1620) at javax.faces.render.Renderer.encodeChildren(Renderer.java:168) at javax.faces.component.UIComponentBase.encodeChildren(UIComponentBase.java:848) at javax.faces.component.UIComponent.encodeAll(UIComponent.java:1613) at javax.faces.component.UIComponent.encodeAll(UIComponent.java:1616) at javax.faces.component.UIComponent.encodeAll(UIComponent.java:1616) at com.sun.faces.application.view.FaceletViewHandlingStrategy.renderView(FaceletViewHandlingStrategy.java:380) at com.sun.faces.application.view.MultiViewHandler.renderView(MultiViewHandler.java:126) at com.sun.faces.lifecycle.RenderResponsePhase.execute(RenderResponsePhase.java:127) at com.sun.faces.lifecycle.Phase.doPhase(Phase.java:101) at com.sun.faces.lifecycle.LifecycleImpl.render(LifecycleImpl.java:139) at javax.faces.webapp.FacesServlet.service(FacesServlet.java:313) at org.apache.catalina.core.StandardWrapper.service(StandardWrapper.java:1523) at org.apache.catalina.core.ApplicationDispatcher.doInvoke(ApplicationDispatcher.java:802) at org.apache.catalina.core.ApplicationDispatcher.invoke(ApplicationDispatcher.java:664) at org.apache.catalina.core.ApplicationDispatcher.processRequest(ApplicationDispatcher.java:497) at org.apache.catalina.core.ApplicationDispatcher.doDispatch(ApplicationDispatcher.java:468) at org.apache.catalina.core.ApplicationDispatcher.dispatch(ApplicationDispatcher.java:364) at org.apache.catalina.core.ApplicationDispatcher.forward(ApplicationDispatcher.java:314) at org.apache.jasper.runtime.PageContextImpl.forward(PageContextImpl.java:783) at org.apache.jsp.welcome_jsp._jspService(welcome_jsp.java from :59) at org.apache.jasper.runtime.HttpJspBase.service(HttpJspBase.java:109) at javax.servlet.http.HttpServlet.service(HttpServlet.java:847) at org.apache.jasper.servlet.JspServletWrapper.service(JspServletWrapper.java:406) at org.apache.jasper.servlet.JspServlet.serviceJspFile(JspServlet.java:483) at org.apache.jasper.servlet.JspServlet.service(JspServlet.java:373) at javax.servlet.http.HttpServlet.service(HttpServlet.java:847) at org.apache.catalina.core.StandardWrapper.service(StandardWrapper.java:1523) at org.apache.catalina.core.StandardWrapperValve.invoke(StandardWrapperValve.java:279) at org.apache.catalina.core.StandardContextValve.invoke(StandardContextValve.java:188) at org.apache.catalina.core.StandardPipeline.invoke(StandardPipeline.java:641) at com.sun.enterprise.web.WebPipeline.invoke(WebPipeline.java:97) at com.sun.enterprise.web.PESessionLockingStandardPipeline.invoke(PESessionLockingStandardPipeline.java:85) at org.apache.catalina.core.StandardHostValve.invoke(StandardHostValve.java:185) at org.apache.catalina.connector.CoyoteAdapter.doService(CoyoteAdapter.java:332) at org.apache.catalina.connector.CoyoteAdapter.service(CoyoteAdapter.java:233) at com.sun.enterprise.v3.services.impl.ContainerMapper.service(ContainerMapper.java:165) at com.sun.grizzly.http.ProcessorTask.invokeAdapter(ProcessorTask.java:791) at com.sun.grizzly.http.ProcessorTask.doProcess(ProcessorTask.java:693) at com.sun.grizzly.http.ProcessorTask.process(ProcessorTask.java:954) at com.sun.grizzly.http.DefaultProtocolFilter.execute(DefaultProtocolFilter.java:170) at com.sun.grizzly.DefaultProtocolChain.executeProtocolFilter(DefaultProtocolChain.java:135) at com.sun.grizzly.DefaultProtocolChain.execute(DefaultProtocolChain.java:102) at com.sun.grizzly.DefaultProtocolChain.execute(DefaultProtocolChain.java:88) at com.sun.grizzly.http.HttpProtocolChain.execute(HttpProtocolChain.java:76) at com.sun.grizzly.ProtocolChainContextTask.doCall(ProtocolChainContextTask.java:53) at com.sun.grizzly.SelectionKeyContextTask.call(SelectionKeyContextTask.java:57) at com.sun.grizzly.ContextTask.run(ContextTask.java:69) at com.sun.grizzly.util.AbstractThreadPool$Worker.doWork(AbstractThreadPool.java:330) at com.sun.grizzly.util.AbstractThreadPool$Worker.run(AbstractThreadPool.java:309) at java.lang.Thread.run(Thread.java:619)

    Read the article

  • CodePlex Daily Summary for Sunday, September 23, 2012

    CodePlex Daily Summary for Sunday, September 23, 2012Popular ReleasesPlayer Framework by Microsoft: Player Framework for Windows 8 (Preview 6): IMPORTANT: List of breaking changes from preview 5 Added separate samples download with .vsix dependencies instead of source dependencies Support for FreeWheel SmartXML ad responses Support for Smooth Streaming SDK DownloaderPlugins Support for VMAP and TTML polling for live scenarios Support for custom smooth streaming byte stream and scheme handlers Support for new play time and position tracking plugin Added IsLiveChanged event Added AdaptivePlugin.MaxBitrate property Add...WPF Application Framework (WAF): WPF Application Framework (WAF) 2.5.0.8: Version: 2.5.0.8 (Milestone 8): This release contains the source code of the WPF Application Framework (WAF) and the sample applications. Requirements .NET Framework 4.0 (The package contains a solution file for Visual Studio 2010) The unit test projects require Visual Studio 2010 Professional Changelog Legend: [B] Breaking change; [O] Marked member as obsolete WAF: Mark the class DataModel as serializable. InfoMan: Minor improvements. InfoMan: Add unit tests for all modules. Othe...LogicCircuit: LogicCircuit 2.12.9.20: Logic Circuit - is educational software for designing and simulating logic circuits. Intuitive graphical user interface, allows you to create unrestricted circuit hierarchy with multi bit buses, debug circuits behavior with oscilloscope, and navigate running circuits hierarchy. Changes of this versionToolbars on text note dialog are more flexible now. You can select font face, size, color, and background of text you are typing. RAM now can be initialized to one of the following: random va...Huo Chess: Huo Chess 0.95: The Huo Chess 0.95 version has an improved chessboard analysis function so as to be able to see which squares are the dangerous squares in the chessboard. This allows the computer to understand better when it is threatened. Two editions are included: Huo Chess 0.95 Console Application (57 KB in size) Huo Chess 0.95 Windows Application with GUI (119 KB in size) See http://harmoniaphilosophica.wordpress.com/2011/09/28/how-to-develop-a-chess-program-for-2jszrulazj6wq-23/ for the infamous How...SiteMap Editor for Microsoft Dynamics CRM 2011: SiteMap Editor (1.1.2020.421): New features: Disable a specific part of SiteMap to keep the data without displaying them in the CRM application. It simply comments XML part of the sitemap (thanks to rboyers for this feature request) Right click an item and click on "Disable" to disable it Items disabled are greyed and a suffix "- disabled" is added Right click an item and click on "Enable" to enable it Refresh list of web resources in the web resources pickerAJAX Control Toolkit: September 2012 Release: AJAX Control Toolkit Release Notes - September 2012 Release Version 60919September 2012 release of the AJAX Control Toolkit. AJAX Control Toolkit .NET 4.5 – AJAX Control Toolkit for .NET 4.5 and sample site (Recommended). AJAX Control Toolkit .NET 4 – AJAX Control Toolkit for .NET 4 and sample site (Recommended). AJAX Control Toolkit .NET 3.5 – AJAX Control Toolkit for .NET 3.5 and sample site (Recommended). Notes: - The current version of the AJAX Control Toolkit is not compatible with ...Sense/Net CMS - Enterprise Content Management: SenseNet 6.1.2 Community Edition: Sense/Net 6.1.2 Community EditionMain new featuresOur current release brings a lot of bugfixes, including the resolution of js/css editing cache issues, xlsx file handling from Office, expense claim demo workspace fixes and much more. Besides fixes 6.1.2 introduces workflow start options and other minor features like a reusable Reject client button for approval scenarios and resource editor enhancements. We have also fixed an issue with our install package to bring you a flawless installation...WinRT XAML Toolkit: WinRT XAML Toolkit - 1.2.3: WinRT XAML Toolkit based on the Windows 8 RTM SDK. Download the latest source from the SOURCE CODE page. For compiled version use NuGet. You can add it to your project in Visual Studio by going to View/Other Windows/Package Manager Console and entering: PM> Install-Package winrtxamltoolkit Features AsyncUI extensions Controls and control extensions Converters Debugging helpers Imaging IO helpers VisualTree helpers Samples Recent changes NOTE: Namespace changes DebugConsol...Python Tools for Visual Studio: 1.5 RC: PTVS 1.5RC Available! We’re pleased to announce the release of Python Tools for Visual Studio 1.5 RC. Python Tools for Visual Studio (PTVS) is an open-source plug-in for Visual Studio which supports programming with the Python language. PTVS supports a broad range of features including CPython/IronPython, Edit/Intellisense/Debug/Profile, Cloud, HPC, IPython, etc. support. The primary new feature for the 1.5 release is Django including Azure support! The http://www.djangoproject.com is a pop...Launchbar: Lanchbar 4.0.0: This application requires .NET 4.5 which you can find here: www.microsoft.com/visualstudio/downloadsAssaultCube Reloaded: 2.5.4 -: Linux has Ubuntu 11.10 32-bit precompiled binaries and Ubuntu 10.10 64-bit precompiled binaries, but you can compile your own as it also contains the source. If you are using Mac or other operating systems, please wait while we try to package for those OSes. Try to compile it. If it fails, download a virtual machine. The server pack is ready for both Windows and Linux, but you might need to compile your own for Linux (source included) Changelog: New logo Improved airstrike! Reset nukes...Extended WPF Toolkit: Extended WPF Toolkit - 1.7.0: Want an easier way to install the Extended WPF Toolkit?The Extended WPF Toolkit is available on Nuget. What's new in the 1.7.0 Release?New controls Zoombox Pie New features / bug fixes PropertyGrid.ShowTitle property added to allow showing/hiding the PropertyGrid title. Modifications to the PropertyGrid.EditorDefinitions collection will now automatically be applied to the PropertyGrid. Modifications to the PropertyGrid.PropertyDefinitions collection will now be reflected automaticaly...JayData - The cross-platform HTML5 data-management library for JavaScript: JayData 1.2: JayData is a unified data access library for JavaScript to CRUD + Query data from different sources like OData, MongoDB, WebSQL, SqLite, Facebook or YQL. The library can be integrated with Knockout.js or Sencha Touch 2 and can be used on Node.js as well. See it in action in this 6 minutes video Sencha Touch 2 example app using JayData: Netflix browser. What's new in JayData 1.2 For detailed release notes check the release notes. JayData core: all async operations now support promises JayDa...LiteBlog (MVC): LiteBlog 1.32: Features added Tree View in Archive widget Upgraded from ASP.NET MVC 3 to MVC 4 Refactored most popular code Added ATOM feed Minor changes to styles????????API for .Net SDK: SDK for .Net ??? Release 4: 2012?9?17??? ?????,???????????????。 ?????Release 3??????,???????,???,??? ??????????????????SDK,????????。 ??,??????? That's all.VidCoder: 1.4.0 Beta: First Beta release! Catches up to HandBrake nightlies with SVN 4937. Added PGS (Blu-ray) subtitle support. Additional framerates available: 30, 50, 59.94, 60 Additional sample rates available: 8, 11.025, 12 and 16 kHz Additional higher bitrates available for audio. Same as Source Constant Framerate available. Added Apple TV 3 preset. Added new Bob deinterlacing option. Introduced process isolation for encodes. Now if HandBrake crashes, VidCoder will keep running and continue pro...DNN Metro7 style Skin package: Metro7 style Skin for DotNetNuke 06.02.01: Stabilization release fixed this issues: Links not worked on FF, Chrome and Safari Modified packaging with own manifest file for install and source package. Moved the user Image on the Login to the left side. Moved h2 font-size to 24px. Note : This release Comes w/o source package about we still work an a solution. Who Needs the Visual Studio source files please go to source and download it from there. Known 16 CSS issues that related to the skin.css. All others are DNN default o...Online Image Editor: Online Image Editor: Features: In this tool, you can edit or adapt your Photo or Image Online in your browser. After uploading, you can adjust your photo by increasing/decreasing Brightness and Contrast. Several filters and effects are available to enhance your photo: Sepia, Sepia and Negative Effect. You can also add Text to your photos and you can choose from any number of common font types. You can adjust text position and color..NET Plugin Manager: 1.0.2012.0917: Provides complete functionality for tiered plugin loading, unloading, and plugin collection management. The Plugin abstract class defines the most primitive plugin requirements and logic. The PluginHost abstract class is a Plugin that loads other plugins. The PluginManager manages the filtering, loading and unloading of plugins. Plugins can be loaded using file path, directory path (with or without recursive directory look-up), and interface type filtering. A Plugin will only be instantia...Free DotNetNuke MultiFunction Skin: MultiFunction Free DotNetNuke Skin v01.02.00: Version 1.2.0 includes the following fixes Removed the clearfix class and is now using dnnClear that ships with DotNetNuke Includes a popupSkin.ascx that is used for the iframe inside the DNN 6 modal windows. Removed control panel DIV tag from all skins as it isn't used in DNN6. Removed left/right borders from the paneOutline class in DNN so that the LAYOUT mode displays without wrapping Had a fix in place for SubMenu CSS when you have a long sub menu Compiled against v6.2.1, requir...New Projects360gu: This project is only used to team build. Alarm Clock: This is a simple free open source MIT licensed alarm clock for Windows. It is less than a hundred lines of code. Written in Visual Studio C# 2010 EE.BlackIce: BlackIce project integrates the MVVM pattern with the MVC pattern providing a set of components that allow you to create a web application from scratch quickly.Casablanca Geodatabase Server: A sample prototype using the Casablanca SDK and the FileGDB API for accessing local geodatabases as GIS service.edx: edexFergo SimpleDXF: A simple DXF library to read layer and entities from DXF files. Easy to use and enough for those who only need the geometry data from the DXF file.Grupa 1 - projekt 1: KoniecIntelligent Bug Tracker System: Intelligent Bug Tracker System a new way to catch bugsKV Player: KV Player has been coded and developed solely by me using Windows Presentation Foundation 4.0. It has been coded in C# 4.0 and the UI has been designed in XAML.MarginCalc: Margin calculator for Windows 8 Metro.MiniProfilerWebFormsEnabler: Sets up MiniProfiler along with a URL you can call to enable/disable the MiniProfiler for your session. Secure File Encrypter: Secure file encrypter is a simple project allowing any user to encrypt or decrypt files on the go! You can use this to basically hide files from anyone!Simple Telnet Client Library: ??????c#?、???telnet?????。suidtool - Batch Matroska Segment UID Reader/Editor: A simple tool/frontend to the mkvtoolnix package to read and/or edit segment UIDs of Matroska files.test project kigod: summary hereUltra Subtitles: This project is registered and implemented as the capstone project for the team members in FPT University.US Elections App for WP7: Source code for the US Elections App (http://www.uselectionsapp.com) for Windows Phone 7. It uses the AgFx, TweetSharp, Northern Lights libsVisual Studio Shell Context Menu: Shell context menu extension for Visual Studio 2010 & 2012.

    Read the article

  • CodePlex Daily Summary for Friday, September 21, 2012

    CodePlex Daily Summary for Friday, September 21, 2012Popular ReleasesWPF Application Framework (WAF): WPF Application Framework (WAF) 2.5.0.8: Version: 2.5.0.8 (Milestone 8): This release contains the source code of the WPF Application Framework (WAF) and the sample applications. Requirements .NET Framework 4.0 (The package contains a solution file for Visual Studio 2010) The unit test projects require Visual Studio 2010 Professional Changelog Legend: [B] Breaking change; [O] Marked member as obsolete WAF: Mark the class DataModel as serializable. InfoMan: Minor improvements. InfoMan: Add unit tests for all modules. Othe...LogicCircuit: LogicCircuit 2.12.9.20: Logic Circuit - is educational software for designing and simulating logic circuits. Intuitive graphical user interface, allows you to create unrestricted circuit hierarchy with multi bit buses, debug circuits behavior with oscilloscope, and navigate running circuits hierarchy. Changes of this versionToolbars on text note dialog are more flexible now. You can select font face, size, color, and background of text you are typing. RAM now can be initialized to one of the following: random va...$linq - A Javascript LINQ library: Version 1.1: Version 1.1 Implemented batch, equiZip, zipLongest, prepend, pad, padWith, toJQuery, pipe, singleOrFallback, indexOf, indexOfElement, lastIndexOf, lastIndexOfElement, scan, prescan, and aggregate operators.Huo Chess: Huo Chess 0.95: The Huo Chess 0.95 version has an improved chessboard analysis function so as to be able to see which squares are the dangerous squares in the chessboard. This allows the computer to understand better when it is threatened. Two editions are included: Huo Chess 0.95 Console Application (57 KB in size) Huo Chess 0.95 Windows Application with GUI (119 KB in size) See http://harmoniaphilosophica.wordpress.com/2011/09/28/how-to-develop-a-chess-program-for-2jszrulazj6wq-23/ for the infamous How...SiteMap Editor for Microsoft Dynamics CRM 2011: SiteMap Editor (1.1.2020.421): New features: Disable a specific part of SiteMap to keep the data without displaying them in the CRM application. It simply comments XML part of the sitemap (thanks to rboyers for this feature request) Right click an item and click on "Disable" to disable it Items disabled are greyed and a suffix "- disabled" is added Right click an item and click on "Enable" to enable it Refresh list of web resources in the web resources pickerAJAX Control Toolkit: September 2012 Release: AJAX Control Toolkit Release Notes - September 2012 Release Version 60919September 2012 release of the AJAX Control Toolkit. AJAX Control Toolkit .NET 4.5 – AJAX Control Toolkit for .NET 4.5 and sample site (Recommended). AJAX Control Toolkit .NET 4 – AJAX Control Toolkit for .NET 4 and sample site (Recommended). AJAX Control Toolkit .NET 3.5 – AJAX Control Toolkit for .NET 3.5 and sample site (Recommended). Notes: - The current version of the AJAX Control Toolkit is not compatible with ...Lib.Web.Mvc & Yet another developer blog: Lib.Web.Mvc 6.1.0: Lib.Web.Mvc is a library which contains some helper classes for ASP.NET MVC such as strongly typed jqGrid helper, XSL transformation HtmlHelper/ActionResult, FileResult with range request support, custom attributes and more. Release contains: Lib.Web.Mvc.dll with xml documentation file Standalone documentation in chm file and change log Library source code Sample application for strongly typed jqGrid helper is available here. Sample application for XSL transformation HtmlHelper/ActionRe...Sense/Net CMS - Enterprise Content Management: SenseNet 6.1.2 Community Edition: Sense/Net 6.1.2 Community EditionMain new featuresOur current release brings a lot of bugfixes, including the resolution of js/css editing cache issues, xlsx file handling from Office, expense claim demo workspace fixes and much more. Besides fixes 6.1.2 introduces workflow start options and other minor features like a reusable Reject client button for approval scenarios and resource editor enhancements. We have also fixed an issue with our install package to bring you a flawless installation...WinRT XAML Toolkit: WinRT XAML Toolkit - 1.2.3: WinRT XAML Toolkit based on the Windows 8 RTM SDK. Download the latest source from the SOURCE CODE page. For compiled version use NuGet. You can add it to your project in Visual Studio by going to View/Other Windows/Package Manager Console and entering: PM> Install-Package winrtxamltoolkit Features AsyncUI extensions Controls and control extensions Converters Debugging helpers Imaging IO helpers VisualTree helpers Samples Recent changes NOTE: Namespace changes DebugConsol...Python Tools for Visual Studio: 1.5 RC: PTVS 1.5RC Available! We’re pleased to announce the release of Python Tools for Visual Studio 1.5 RC. Python Tools for Visual Studio (PTVS) is an open-source plug-in for Visual Studio which supports programming with the Python language. PTVS supports a broad range of features including CPython/IronPython, Edit/Intellisense/Debug/Profile, Cloud, HPC, IPython, etc. support. The primary new feature for the 1.5 release is Django including Azure support! The http://www.djangoproject.com is a pop...Launchbar: Lanchbar 4.0.0: First public release.AssaultCube Reloaded: 2.5.4 -: Linux has Ubuntu 11.10 32-bit precompiled binaries and Ubuntu 10.10 64-bit precompiled binaries, but you can compile your own as it also contains the source. If you are using Mac or other operating systems, please wait while we try to package for those OSes. Try to compile it. If it fails, download a virtual machine. The server pack is ready for both Windows and Linux, but you might need to compile your own for Linux (source included) Changelog: New logo Improved airstrike! Reset nukes...Extended WPF Toolkit: Extended WPF Toolkit - 1.7.0: Want an easier way to install the Extended WPF Toolkit?The Extended WPF Toolkit is available on Nuget. What's new in the 1.7.0 Release?New controls Zoombox Pie New features / bug fixes PropertyGrid.ShowTitle property added to allow showing/hiding the PropertyGrid title. Modifications to the PropertyGrid.EditorDefinitions collection will now automatically be applied to the PropertyGrid. Modifications to the PropertyGrid.PropertyDefinitions collection will now be reflected automaticaly...JayData - The cross-platform HTML5 data-management library for JavaScript: JayData 1.2: JayData is a unified data access library for JavaScript to CRUD + Query data from different sources like OData, MongoDB, WebSQL, SqLite, Facebook or YQL. The library can be integrated with Knockout.js or Sencha Touch 2 and can be used on Node.js as well. See it in action in this 6 minutes video Sencha Touch 2 example app using JayData: Netflix browser. What's new in JayData 1.2 For detailed release notes check the release notes. JayData core: all async operations now support promises JayDa...????????API for .Net SDK: SDK for .Net ??? Release 4: 2012?9?17??? ?????,???????????????。 ?????Release 3??????,???????,???,??? ??????????????????SDK,????????。 ??,??????? That's all.VidCoder: 1.4.0 Beta: First Beta release! Catches up to HandBrake nightlies with SVN 4937. Added PGS (Blu-ray) subtitle support. Additional framerates available: 30, 50, 59.94, 60 Additional sample rates available: 8, 11.025, 12 and 16 kHz Additional higher bitrates available for audio. Same as Source Constant Framerate available. Added Apple TV 3 preset. Added new Bob deinterlacing option. Introduced process isolation for encodes. Now if HandBrake crashes, VidCoder will keep running and continue pro...DNN Metro7 style Skin package: Metro7 style Skin for DotNetNuke 06.02.01: Stabilization release fixed this issues: Links not worked on FF, Chrome and Safari Modified packaging with own manifest file for install and source package. Moved the user Image on the Login to the left side. Moved h2 font-size to 24px. Note : This release Comes w/o source package about we still work an a solution. Who Needs the Visual Studio source files please go to source and download it from there. Known 16 CSS issues that related to the skin.css. All others are DNN default o...Visual Studio Icon Patcher: Version 1.5.1: This fixes a bug in the 1.5 release where it would crash when no language packs were installed for VS2010.VFPX: Desktop Alerts 1.0.2: This update for the Desktop Alerts contains changes to behavior for setting custom sounds for alerts. I have removed ALERTWAV.TXT from the project, and also removed DA_DEFAULTSOUND from the VFPALERT.H file. The AlertManager class and Alert class both have a "default" cSound of ADDBS(JUSTPATH(_VFP.ServerName))+"alert.wav" --- so, as long as you distribute a sound file with the file name "alert.wav" along with the EXE, that file will be used. You can set your own sound file globally by setti...MCEBuddy 2.x: MCEBuddy 2.2.15: Changelog for 2.2.15 (32bit and 64bit) 1. Added support for %originalfilepath% to get the source file full path. Used for custom commands only. 2. Added support for better parsing of Media Portal XML files to extract ShowName and Episode Name and download additional details from TVDB (like Season No, Episode No etc). 3. Added support for TVDB seriesID in metadata 4. Added support for eMail non blocking UI testNew ProjectsAppDevPoint: Cross-platform development tool for native applications without rewriting the business logic code but maintaining the speed and appearance of the OS.Author-it Plugin Hello World: A simple plug-in to help developers understand how to create a plug-in for Author-it. Build SharePoint Applications with Windows 8: Create a SharePoint Lists reader for Windows Store apps using C#/VB and XAML via WCFCatwitter - a WinRT example with Twitter and Catel: WinRT example of Catel in combination with Twitter.Community xPress MDS: Community xPress MDS is intended to serve as a reference Master Data Management implementation and learning tool for those interested in MDS & DQS.ControlDesktop: My LoveDITA to Author-it Plugin: This project is an Author-it plug-in that allows you to import several files types into Author-it, including DITA. EasyPrint: Straight forward class used to print text documents in Embarcadero's C++ Builder. Wraps the TPrinter object to allow a programmer to print pages of text.Ffmpeg converter: This is a tool that monitors folders and that converts then to the desired format. Feel free to contributeHCIProject: HCI tantárgy házi feladata, mozimusor alkalmazás.Keyboard capture of F1 and F2 (and others) in Compact Framework: How to capture F1 and F2 in Compact Framework on Windows Embedded Handheld 6.5.3MCSave: Automatically "reload" to a previous point your Minecraft game. Lose all your diamonds to that conveniently place pool of lava? Not anymore.Microsoft Dynamics CRM 2011 Event Binder: Bind form events for CRM 2011 entities without drilling into a CRM form entity. An easier way to manage CRM jJavascript events.Mini Author-it Exporter: This is a minimal console application written in .NET/C# that exports a topic from Author-it, displays it, then closes. Mini Author-it Importer: A minimal importer for the Author-it system.MyMediaStore: Sistema para registro de mídias de acervo no computador. Versão atual: 1.0.2 - RTM. Nesta versão: Registro de vídeos. Visualização de vídeos. NAV Metro: Dynamics NAV 2013 Metro App framework.netcloud: dfdfPlanar Mechanism Kinematic Simulator (PMKS): PMKS returns quick and accurate results for the position, velocity, and acceleration of rigid bodies connected as planar mechanisms.Project Austin: Austin is a digital note-taking app for Windows 8.robintools: nothing is doingSalud Ocupacional: salud ocupacionalText-Based Calculator: Text-based calculator that accepts a string of input and performs mathematical operations on the numeric values represented by the data in the string. Tôi di h?i l?: This project only have Vietnamese version and follow function from I Paid a Bribe.comtuXXdo: This is extended from Orchard blog engine.Xiaoweiyu toolkit: toolsXiaoweiyu website: just a test

    Read the article

  • CodePlex Daily Summary for Saturday, September 22, 2012

    CodePlex Daily Summary for Saturday, September 22, 2012Popular ReleasesWPF Application Framework (WAF): WPF Application Framework (WAF) 2.5.0.8: Version: 2.5.0.8 (Milestone 8): This release contains the source code of the WPF Application Framework (WAF) and the sample applications. Requirements .NET Framework 4.0 (The package contains a solution file for Visual Studio 2010) The unit test projects require Visual Studio 2010 Professional Changelog Legend: [B] Breaking change; [O] Marked member as obsolete WAF: Mark the class DataModel as serializable. InfoMan: Minor improvements. InfoMan: Add unit tests for all modules. Othe...LogicCircuit: LogicCircuit 2.12.9.20: Logic Circuit - is educational software for designing and simulating logic circuits. Intuitive graphical user interface, allows you to create unrestricted circuit hierarchy with multi bit buses, debug circuits behavior with oscilloscope, and navigate running circuits hierarchy. Changes of this versionToolbars on text note dialog are more flexible now. You can select font face, size, color, and background of text you are typing. RAM now can be initialized to one of the following: random va...$linq - A Javascript LINQ library: Version 1.1: Version 1.1 Implemented batch, equiZip, zipLongest, prepend, pad, padWith, toJQuery, pipe, singleOrFallback, indexOf, indexOfElement, lastIndexOf, lastIndexOfElement, scan, prescan, and aggregate operators.Huo Chess: Huo Chess 0.95: The Huo Chess 0.95 version has an improved chessboard analysis function so as to be able to see which squares are the dangerous squares in the chessboard. This allows the computer to understand better when it is threatened. Two editions are included: Huo Chess 0.95 Console Application (57 KB in size) Huo Chess 0.95 Windows Application with GUI (119 KB in size) See http://harmoniaphilosophica.wordpress.com/2011/09/28/how-to-develop-a-chess-program-for-2jszrulazj6wq-23/ for the infamous How...Symphony Framework: Symphony Framework v2.0.0.2: Symphony Framework version 2.0.0.2. General note: If you install Symphony Framework 2.0.0.2 you must also install CodeGen 4.1.10 because a number of templates now utilise new features added to the tool. Added the user token PROJECTNAMESPACE to the “Symphony_Content.tpl” template to ensure that we can correctly reference the collection classes of the selection lists. Also added the ability to create object references to fields defined as having selection windows assigned. This enhancement ...Community xPress MDS: Initial MDS and DQS Models: Initial MDS & DQS ModelsSiteMap Editor for Microsoft Dynamics CRM 2011: SiteMap Editor (1.1.2020.421): New features: Disable a specific part of SiteMap to keep the data without displaying them in the CRM application. It simply comments XML part of the sitemap (thanks to rboyers for this feature request) Right click an item and click on "Disable" to disable it Items disabled are greyed and a suffix "- disabled" is added Right click an item and click on "Enable" to enable it Refresh list of web resources in the web resources pickerAJAX Control Toolkit: September 2012 Release: AJAX Control Toolkit Release Notes - September 2012 Release Version 60919September 2012 release of the AJAX Control Toolkit. AJAX Control Toolkit .NET 4.5 – AJAX Control Toolkit for .NET 4.5 and sample site (Recommended). AJAX Control Toolkit .NET 4 – AJAX Control Toolkit for .NET 4 and sample site (Recommended). AJAX Control Toolkit .NET 3.5 – AJAX Control Toolkit for .NET 3.5 and sample site (Recommended). Notes: - The current version of the AJAX Control Toolkit is not compatible with ...Lib.Web.Mvc & Yet another developer blog: Lib.Web.Mvc 6.1.0: Lib.Web.Mvc is a library which contains some helper classes for ASP.NET MVC such as strongly typed jqGrid helper, XSL transformation HtmlHelper/ActionResult, FileResult with range request support, custom attributes and more. Release contains: Lib.Web.Mvc.dll with xml documentation file Standalone documentation in chm file and change log Library source code Sample application for strongly typed jqGrid helper is available here. Sample application for XSL transformation HtmlHelper/ActionRe...Sense/Net CMS - Enterprise Content Management: SenseNet 6.1.2 Community Edition: Sense/Net 6.1.2 Community EditionMain new featuresOur current release brings a lot of bugfixes, including the resolution of js/css editing cache issues, xlsx file handling from Office, expense claim demo workspace fixes and much more. Besides fixes 6.1.2 introduces workflow start options and other minor features like a reusable Reject client button for approval scenarios and resource editor enhancements. We have also fixed an issue with our install package to bring you a flawless installation...WinRT XAML Toolkit: WinRT XAML Toolkit - 1.2.3: WinRT XAML Toolkit based on the Windows 8 RTM SDK. Download the latest source from the SOURCE CODE page. For compiled version use NuGet. You can add it to your project in Visual Studio by going to View/Other Windows/Package Manager Console and entering: PM> Install-Package winrtxamltoolkit Features AsyncUI extensions Controls and control extensions Converters Debugging helpers Imaging IO helpers VisualTree helpers Samples Recent changes NOTE: Namespace changes DebugConsol...Python Tools for Visual Studio: 1.5 RC: PTVS 1.5RC Available! We’re pleased to announce the release of Python Tools for Visual Studio 1.5 RC. Python Tools for Visual Studio (PTVS) is an open-source plug-in for Visual Studio which supports programming with the Python language. PTVS supports a broad range of features including CPython/IronPython, Edit/Intellisense/Debug/Profile, Cloud, HPC, IPython, etc. support. The primary new feature for the 1.5 release is Django including Azure support! The http://www.djangoproject.com is a pop...Launchbar: Lanchbar 4.0.0: This application requires .NET 4.5 which you can find here: www.microsoft.com/visualstudio/downloadsAssaultCube Reloaded: 2.5.4 -: Linux has Ubuntu 11.10 32-bit precompiled binaries and Ubuntu 10.10 64-bit precompiled binaries, but you can compile your own as it also contains the source. If you are using Mac or other operating systems, please wait while we try to package for those OSes. Try to compile it. If it fails, download a virtual machine. The server pack is ready for both Windows and Linux, but you might need to compile your own for Linux (source included) Changelog: New logo Improved airstrike! Reset nukes...Extended WPF Toolkit: Extended WPF Toolkit - 1.7.0: Want an easier way to install the Extended WPF Toolkit?The Extended WPF Toolkit is available on Nuget. What's new in the 1.7.0 Release?New controls Zoombox Pie New features / bug fixes PropertyGrid.ShowTitle property added to allow showing/hiding the PropertyGrid title. Modifications to the PropertyGrid.EditorDefinitions collection will now automatically be applied to the PropertyGrid. Modifications to the PropertyGrid.PropertyDefinitions collection will now be reflected automaticaly...JayData - The cross-platform HTML5 data-management library for JavaScript: JayData 1.2: JayData is a unified data access library for JavaScript to CRUD + Query data from different sources like OData, MongoDB, WebSQL, SqLite, Facebook or YQL. The library can be integrated with Knockout.js or Sencha Touch 2 and can be used on Node.js as well. See it in action in this 6 minutes video Sencha Touch 2 example app using JayData: Netflix browser. What's new in JayData 1.2 For detailed release notes check the release notes. JayData core: all async operations now support promises JayDa...????????API for .Net SDK: SDK for .Net ??? Release 4: 2012?9?17??? ?????,???????????????。 ?????Release 3??????,???????,???,??? ??????????????????SDK,????????。 ??,??????? That's all.VidCoder: 1.4.0 Beta: First Beta release! Catches up to HandBrake nightlies with SVN 4937. Added PGS (Blu-ray) subtitle support. Additional framerates available: 30, 50, 59.94, 60 Additional sample rates available: 8, 11.025, 12 and 16 kHz Additional higher bitrates available for audio. Same as Source Constant Framerate available. Added Apple TV 3 preset. Added new Bob deinterlacing option. Introduced process isolation for encodes. Now if HandBrake crashes, VidCoder will keep running and continue pro...DNN Metro7 style Skin package: Metro7 style Skin for DotNetNuke 06.02.01: Stabilization release fixed this issues: Links not worked on FF, Chrome and Safari Modified packaging with own manifest file for install and source package. Moved the user Image on the Login to the left side. Moved h2 font-size to 24px. Note : This release Comes w/o source package about we still work an a solution. Who Needs the Visual Studio source files please go to source and download it from there. Known 16 CSS issues that related to the skin.css. All others are DNN default o...Visual Studio Icon Patcher: Version 1.5.1: This fixes a bug in the 1.5 release where it would crash when no language packs were installed for VS2010.New ProjectsCodePlexDeployment: Please ignore, this project is for testing out some features of the WAWS deployment integrationDotNetNuke Social Dashboard: The DotNetNuke Social Dashboard gives DotNetNuke Administrators and insight into the social statistics of their site.EESTEC LC Trieste: .Event Log Mailer: Mails events from Windows' system Event Log which matches rules in configuration. Runs as Windows service and has super simple configurationflx4432: Lorem ipsum dolor sit amet, consectetur adipiscing elit. Aliquam facilisis condimentum nulla. Duis sed quam vitae nunc semper facilisis a eget leo.lanWOLf: Send wake-on-lan packets across subnets by utilizing powered-on machines on each subnet.Micro-Apps Framework: Micro-Apps is a revolutionary piece of software that allows you to have multiple programs running from 1 file under the same process!NJ: NJ Language Learning Helper type Config() = // Just Code Example member x.GetAll() = seq{ yield {Name="Admin" Dictionary = OtfPG: To generate reproducible complex passwords from simple pass phrases, allowing the user to 'remember; a simple phrase, rather that a complex password, without evProject92104: as ppProject92105: ppaProject92107: papaPython intellisense Enhancer: For the python code, the intellisense box will show after you input a character, just like c#.QR Code Reader (By Screen Capture): Reads the QR Codes displayed in webpages. (You need to capture the code area) Displays the code information.scenariov1706jabbr: helloSharePoint 2013 REST Test Web Part: A simple web part (placed in a farm solution) that helps SharePoint developers to test every HTTP call to the new REST interface of SharePoint 2013.SharePoint Resources Updater (2010 /2013): Project for SharePoint 2010/2013 IT pro's and dev's to adress App_GlobalResources difficulties when developping SharePoint solutions(or maintaining large farms)Word CustomXML data services: Services to add,change and read metadata embedded into a Word document. Metadata are stored in a custom XML file into the Word document. ??????: ge ren xiang mu

    Read the article

  • Top 50 ASP.Net Interview Questions & Answers

    - by Samir R. Bhogayta
    1. What is ASP.Net? It is a framework developed by Microsoft on which we can develop new generation web sites using web forms(aspx), MVC, HTML, Javascript, CSS etc. Its successor of Microsoft Active Server Pages(ASP). Currently there is ASP.NET 4.0, which is used to develop web sites. There are various page extensions provided by Microsoft that are being used for web site development. Eg: aspx, asmx, ascx, ashx, cs, vb, html, xml etc. 2. What’s the use of Response.Output.Write()? We can write formatted output  using Response.Output.Write(). 3. In which event of page cycle is the ViewState available?   After the Init() and before the Page_Load(). 4. What is the difference between Server.Transfer and Response.Redirect?   In Server.Transfer page processing transfers from one page to the other page without making a round-trip back to the client’s browser.  This provides a faster response with a little less overhead on the server.  The clients url history list or current url Server does not update in case of Server.Transfer. Response.Redirect is used to redirect the user’s browser to another page or site.  It performs trip back to the client where the client’s browser is redirected to the new page.  The user’s browser history list is updated to reflect the new address. 5. From which base class all Web Forms are inherited? Page class.  6. What are the different validators in ASP.NET? Required field Validator Range  Validator Compare Validator Custom Validator Regular expression Validator Summary Validator 7. Which validator control you use if you need to make sure the values in two different controls matched? Compare Validator control. 8. What is ViewState? ViewState is used to retain the state of server-side objects between page post backs. 9. Where the viewstate is stored after the page postback? ViewState is stored in a hidden field on the page at client side.  ViewState is transported to the client and back to the server, and is not stored on the server or any other external source. 10. How long the items in ViewState exists? They exist for the life of the current page. 11. What are the different Session state management options available in ASP.NET? In-Process Out-of-Process. In-Process stores the session in memory on the web server. Out-of-Process Session state management stores data in an external server.  The external server may be either a SQL Server or a State Server.  All objects stored in session are required to be serializable for Out-of-Process state management. 12. How you can add an event handler?  Using the Attributes property of server side control. e.g. [csharp] btnSubmit.Attributes.Add(“onMouseOver”,”JavascriptCode();”) [/csharp] 13. What is caching? Caching is a technique used to increase performance by keeping frequently accessed data or files in memory. The request for a cached file/data will be accessed from cache instead of actual location of that file. 14. What are the different types of caching? ASP.NET has 3 kinds of caching : Output Caching, Fragment Caching, Data Caching. 15. Which type if caching will be used if we want to cache the portion of a page instead of whole page? Fragment Caching: It caches the portion of the page generated by the request. For that, we can create user controls with the below code: [xml] <%@ OutputCache Duration=”120? VaryByParam=”CategoryID;SelectedID”%> [/xml] 16. List the events in page life cycle.   1) Page_PreInit 2) Page_Init 3) Page_InitComplete 4) Page_PreLoad 5) Page_Load 6) Page_LoadComplete 7) Page_PreRender 8)Render 17. Can we have a web application running without web.Config file?   Yes 18. Is it possible to create web application with both webforms and mvc? Yes. We have to include below mvc assembly references in the web forms application to create hybrid application. [csharp] System.Web.Mvc System.Web.Razor System.ComponentModel.DataAnnotations [/csharp] 19. Can we add code files of different languages in App_Code folder?   No. The code files must be in same language to be kept in App_code folder. 20. What is Protected Configuration? It is a feature used to secure connection string information. 21. Write code to send e-mail from an ASP.NET application? [csharp] MailMessage mailMess = new MailMessage (); mailMess.From = “[email protected]”; mailMess.To = “[email protected]”; mailMess.Subject = “Test email”; mailMess.Body = “Hi This is a test mail.”; SmtpMail.SmtpServer = “localhost”; SmtpMail.Send (mailMess); [/csharp] MailMessage and SmtpMail are classes defined System.Web.Mail namespace.  22. How can we prevent browser from caching an ASPX page?   We can SetNoStore on HttpCachePolicy object exposed by the Response object’s Cache property: [csharp] Response.Cache.SetNoStore (); Response.Write (DateTime.Now.ToLongTimeString ()); [/csharp] 23. What is the good practice to implement validations in aspx page? Client-side validation is the best way to validate data of a web page. It reduces the network traffic and saves server resources. 24. What are the event handlers that we can have in Global.asax file? Application Events: Application_Start , Application_End, Application_AcquireRequestState, Application_AuthenticateRequest, Application_AuthorizeRequest, Application_BeginRequest, Application_Disposed,  Application_EndRequest, Application_Error, Application_PostRequestHandlerExecute, Application_PreRequestHandlerExecute, Application_PreSendRequestContent, Application_PreSendRequestHeaders, Application_ReleaseRequestState, Application_ResolveRequestCache, Application_UpdateRequestCache Session Events: Session_Start,Session_End 25. Which protocol is used to call a Web service? HTTP Protocol 26. Can we have multiple web config files for an asp.net application? Yes. 27. What is the difference between web config and machine config? Web config file is specific to a web application where as machine config is specific to a machine or server. There can be multiple web config files into an application where as we can have only one machine config file on a server. 28.  Explain role based security ?   Role Based Security used to implement security based on roles assigned to user groups in the organization. Then we can allow or deny users based on their role in the organization. Windows defines several built-in groups, including Administrators, Users, and Guests. [xml] <AUTHORIZATION>< authorization > < allow roles=”Domain_Name\Administrators” / >   < !– Allow Administrators in domain. — > < deny users=”*”  / >                            < !– Deny anyone else. — > < /authorization > [/xml] 29. What is Cross Page Posting? When we click submit button on a web page, the page post the data to the same page. The technique in which we post the data to different pages is called Cross Page posting. This can be achieved by setting POSTBACKURL property of  the button that causes the postback. Findcontrol method of PreviousPage can be used to get the posted values on the page to which the page has been posted. 30. How can we apply Themes to an asp.net application? We can specify the theme in web.config file. Below is the code example to apply theme: [xml] <configuration> <system.web> <pages theme=”Windows7? /> </system.web> </configuration> [/xml] 31: What is RedirectPermanent in ASP.Net?   RedirectPermanent Performs a permanent redirection from the requested URL to the specified URL. Once the redirection is done, it also returns 301 Moved Permanently responses. 32: What is MVC? MVC is a framework used to create web applications. The web application base builds on  Model-View-Controller pattern which separates the application logic from UI, and the input and events from the user will be controlled by the Controller. 33. Explain the working of passport authentication. First of all it checks passport authentication cookie. If the cookie is not available then the application redirects the user to Passport Sign on page. Passport service authenticates the user details on sign on page and if valid then stores the authenticated cookie on client machine and then redirect the user to requested page 34. What are the advantages of Passport authentication? All the websites can be accessed using single login credentials. So no need to remember login credentials for each web site. Users can maintain his/ her information in a single location. 35. What are the asp.net Security Controls? <asp:Login>: Provides a standard login capability that allows the users to enter their credentials <asp:LoginName>: Allows you to display the name of the logged-in user <asp:LoginStatus>: Displays whether the user is authenticated or not <asp:LoginView>: Provides various login views depending on the selected template <asp:PasswordRecovery>:  email the users their lost password 36: How do you register JavaScript for webcontrols ? We can register javascript for controls using <CONTROL -name>Attribtues.Add(scriptname,scripttext) method. 37. In which event are the controls fully loaded? Page load event. 38: what is boxing and unboxing? Boxing is assigning a value type to reference type variable. Unboxing is reverse of boxing ie. Assigning reference type variable to value type variable. 39. Differentiate strong typing and weak typing In strong typing, the data types of variable are checked at compile time. On the other hand, in case of weak typing the variable data types are checked at runtime. In case of strong typing, there is no chance of compilation error. Scripts use weak typing and hence issues arises at runtime. 40. How we can force all the validation controls to run? The Page.Validate() method is used to force all the validation controls to run and to perform validation. 41. List all templates of the Repeater control. ItemTemplate AlternatingltemTemplate SeparatorTemplate HeaderTemplate FooterTemplate 42. List the major built-in objects in ASP.NET?  Application Request Response Server Session Context Trace 43. What is the appSettings Section in the web.config file? The appSettings block in web config file sets the user-defined values for the whole application. For example, in the following code snippet, the specified ConnectionString section is used throughout the project for database connection: [csharp] <em><configuration> <appSettings> <add key=”ConnectionString” value=”server=local; pwd=password; database=default” /> </appSettings></em> [/csharp] 44.      Which data type does the RangeValidator control support? The data types supported by the RangeValidator control are Integer, Double, String, Currency, and Date. 45. What is the difference between an HtmlInputCheckBox control and anHtmlInputRadioButton control? In HtmlInputCheckBoxcontrol, multiple item selection is possible whereas in HtmlInputRadioButton controls, we can select only single item from the group of items. 46. Which namespaces are necessary to create a localized application? System.Globalization System.Resources 47. What are the different types of cookies in ASP.NET? Session Cookie – Resides on the client machine for a single session until the user does not log out. Persistent Cookie – Resides on a user’s machine for a period specified for its expiry, such as 10 days, one month, and never. 48. What is the file extension of web service? Web services have file extension .asmx.. 49. What are the components of ADO.NET? The components of ADO.Net are Dataset, Data Reader, Data Adaptor, Command, connection. 50. What is the difference between ExecuteScalar and ExecuteNonQuery? ExecuteScalar returns output value where as ExecuteNonQuery does not return any value but the number of rows affected by the query. ExecuteScalar used for fetching a single value and ExecuteNonQuery used to execute Insert and Update statements.

    Read the article

  • JSF 2.1 Spring 3.0 Integration

    - by danny.lesnik
    I'm trying to make very simple Spring 3 + JSF2.1 integration according to examples I googled in the web. So here is my code: My HTML submitted to actionController.actionSubmitted() method: <h:form> <h:message for="textPanel" style="color:red;" /> <h:panelGrid columns="3" rows="5" id="textPanel"> //all my bean prperties mapped to HTML code. </h:panelGrid> <h:commandButton value="Submit" action="#{actionController.actionSubmitted}" /> </h:form> now the Action Controller itself: @ManagedBean(name="actionController") @SessionScoped public class ActionController implements Serializable{ @ManagedProperty(value="#{user}") User user; @ManagedProperty(value="#{mailService}") MailService mailService; public void setMailService(MailService mailService) { this.mailService = mailService; } public void setUser(User user) { this.user = user; } private static final long serialVersionUID = 1L; public ActionController() {} public String actionSubmitted(){ System.out.println(user.getEmail()); mailService.sendUserMail(user); return "success"; } } Now my bean Spring: public interface MailService { void sendUserMail(User user); } public class MailServiceImpl implements MailService{ @Override public void sendUserMail(User user) { System.out.println("Mail to "+user.getEmail()+" sent." ); } } This is my web.xml <listener> <listener-class> org.springframework.web.context.ContextLoaderListener </listener-class> </listener> <listener> <listener-class> org.springframework.web.context.request.RequestContextListener </listener-class> </listener> <!-- Welcome page --> <welcome-file-list> <welcome-file>index.xhtml</welcome-file> </welcome-file-list> <!-- JSF mapping --> <servlet> <servlet-name>Faces Servlet</servlet-name> <servlet-class>javax.faces.webapp.FacesServlet</servlet-class> <load-on-startup>1</load-on-startup> </servlet> my applicationContext.xml <beans xmlns="http://www.springframework.org/schema/beans" xmlns:xsi="http://www.w3.org/2001/XMLSchema-instance" xsi:schemaLocation="http://www.springframework.org/schema/beans http://www.springframework.org/schema/beans/spring-beans-3.0.xsd"> <bean id="mailService" class="com.vanilla.jsf.services.MailServiceImpl"> </bean> </beans> my faces-config.xml is the following: <application> <el-resolver> org.springframework.web.jsf.el.SpringBeanFacesELResolver </el-resolver> <message-bundle> com.vanilla.jsf.validators.MyMessages </message-bundle> </application> <managed-bean> <managed-bean-name>actionController</managed-bean-name> <managed-bean-class>com.vanilla.jsf.controllers.ActionController</managed-bean-class> <managed-bean-scope>session</managed-bean-scope> <managed-property> <property-name>mailService</property-name> <value>#{mailService}</value> </managed-property> </managed-bean> <navigation-rule> <from-view-id>index.xhtml</from-view-id> <navigation-case> <from-action>#{actionController.actionSubmitted}</from-action> <from-outcome>success</from-outcome> <to-view-id>submitted.xhtml</to-view-id> <redirect /> </navigation-case> </navigation-rule> My Problem is that I'm getting NullPointerExeption because my mailService Spring bean is null. public String actionSubmitted(){ System.out.println(user.getEmail()); //mailService is null Getting NullPointerException mailService.sendUserMail(user); return "success"; }

    Read the article

  • Get the property, as a string, from an Expression<Func<TModel,TProperty>>

    - by Jaxidian
    I use some strongly-typed expressions that get serialized to allow my UI code to have strongly-typed sorting and searching expressions. These are of type Expression<Func<TModel,TProperty>> and are used as such: SortOption.Field = (p => p.FirstName);. I've gotten this working perfectly for this simple case. The code that I'm using for parsing the "FirstName" property out of there is actually reusing some existing functionality in a third-party product that we use and it works great, until we start working with deeply-nested properties(SortOption.Field = (p => p.Address.State.Abbreviation);). This code has some very different assumptions in the need to support deeply-nested properties. As for what this code does, I don't really understand it and rather than changing that code, I figured I should just write from scratch this functionality. However, I don't know of a good way to do this. I suspect we can do something better than doing a ToString() and performing string parsing. So what's a good way to do this to handle the trivial and deeply-nested cases? Requirements: Given the expression p => p.FirstName I need a string of "FirstName". Given the expression p => p.Address.State.Abbreviation I need a string of "Address.State.Abbreviation" While it's not important for an answer to my question, I suspect my serialization/deserialization code could be useful to somebody else who finds this question in the future, so it is below. Again, this code is not important to the question - I just thought it might help somebody. Note that DynamicExpression.ParseLambda comes from the Dynamic LINQ stuff and Property.PropertyToString() is what this question is about. /// <summary> /// This defines a framework to pass, across serialized tiers, sorting logic to be performed. /// </summary> /// <typeparam name="TModel">This is the object type that you are filtering.</typeparam> /// <typeparam name="TProperty">This is the property on the object that you are filtering.</typeparam> [Serializable] public class SortOption<TModel, TProperty> : ISerializable where TModel : class { /// <summary> /// Convenience constructor. /// </summary> /// <param name="property">The property to sort.</param> /// <param name="isAscending">Indicates if the sorting should be ascending or descending</param> /// <param name="priority">Indicates the sorting priority where 0 is a higher priority than 10.</param> public SortOption(Expression<Func<TModel, TProperty>> property, bool isAscending = true, int priority = 0) { Property = property; IsAscending = isAscending; Priority = priority; } /// <summary> /// Default Constructor. /// </summary> public SortOption() : this(null) { } /// <summary> /// This is the field on the object to filter. /// </summary> public Expression<Func<TModel, TProperty>> Property { get; set; } /// <summary> /// This indicates if the sorting should be ascending or descending. /// </summary> public bool IsAscending { get; set; } /// <summary> /// This indicates the sorting priority where 0 is a higher priority than 10. /// </summary> public int Priority { get; set; } #region Implementation of ISerializable /// <summary> /// This is the constructor called when deserializing a SortOption. /// </summary> protected SortOption(SerializationInfo info, StreamingContext context) { IsAscending = info.GetBoolean("IsAscending"); Priority = info.GetInt32("Priority"); // We just persisted this by the PropertyName. So let's rebuild the Lambda Expression from that. Property = DynamicExpression.ParseLambda<TModel, TProperty>(info.GetString("Property"), default(TModel), default(TProperty)); } /// <summary> /// Populates a <see cref="T:System.Runtime.Serialization.SerializationInfo"/> with the data needed to serialize the target object. /// </summary> /// <param name="info">The <see cref="T:System.Runtime.Serialization.SerializationInfo"/> to populate with data. </param> /// <param name="context">The destination (see <see cref="T:System.Runtime.Serialization.StreamingContext"/>) for this serialization. </param> public void GetObjectData(SerializationInfo info, StreamingContext context) { // Just stick the property name in there. We'll rebuild the expression based on that on the other end. info.AddValue("Property", Property.PropertyToString()); info.AddValue("IsAscending", IsAscending); info.AddValue("Priority", Priority); } #endregion }

    Read the article

  • Collections not read from hibernate/ehcache second-level-cache

    - by Mark van Venrooij
    I'm trying to cache lazy loaded collections with ehcache/hibernate in a Spring project. When I execute a session.get(Parent.class, 123) and browse through the children multiple times a query is executed every time to fetch the children. The parent is only queried the first time and then resolved from the cache. Probably I'm missing something, but I can't find the solution. Please see the relevant code below. I'm using Spring (3.2.4.RELEASE) Hibernate(4.2.1.Final) and ehcache(2.6.6) The parent class: @Entity @Table(name = "PARENT") @Cacheable @Cache(usage = CacheConcurrencyStrategy.READ_WRITE, include = "all") public class ServiceSubscriptionGroup implements Serializable { /** The Id. */ @Id @Column(name = "ID") private int id; @OneToMany(fetch = FetchType.LAZY, mappedBy = "parent") @Cache(usage = CacheConcurrencyStrategy.READ_WRITE) private List<Child> children; public List<Child> getChildren() { return children; } public void setChildren(List<Child> children) { this.children = children; } @Override public boolean equals(Object o) { if (this == o) return true; if (o == null || getClass() != o.getClass()) return false; Parent that = (Parent) o; if (id != that.id) return false; return true; } @Override public int hashCode() { return id; } } The child class: @Entity @Table(name = "CHILD") @Cacheable @Cache(usage = CacheConcurrencyStrategy.READ_WRITE, include = "all") public class Child { @Id @Column(name = "ID") private int id; @ManyToOne(fetch = FetchType.LAZY, cascade = CascadeType.ALL) @JoinColumn(name = "PARENT_ID") @Cache(usage = CacheConcurrencyStrategy.READ_WRITE) private Parent parent; public int getId() { return id; } public void setId(final int id) { this.id = id; } private Parent getParent(){ return parent; } private void setParent(Parent parent) { this.parent = parent; } @Override public boolean equals(final Object o) { if (this == o) { return true; } if (o == null || getClass() != o.getClass()) { return false; } final Child that = (Child) o; return id == that.id; } @Override public int hashCode() { return id; } } The application context: <bean id="sessionFactory" class="org.springframework.orm.hibernate4.LocalSessionFactoryBean"> <property name="dataSource" ref="dataSource" /> <property name="annotatedClasses"> <list> <value>Parent</value> <value>Child</value> </list> </property> <property name="hibernateProperties"> <props> <prop key="hibernate.dialect">org.hibernate.dialect.SQLServer2008Dialect</prop> <prop key="hibernate.hbm2ddl.auto">validate</prop> <prop key="hibernate.ejb.naming_strategy">org.hibernate.cfg.ImprovedNamingStrategy</prop> <prop key="hibernate.connection.charSet">UTF-8</prop> <prop key="hibernate.show_sql">true</prop> <prop key="hibernate.format_sql">true</prop> <prop key="hibernate.use_sql_comments">true</prop> <!-- cache settings ehcache--> <prop key="hibernate.cache.use_second_level_cache">true</prop> <prop key="hibernate.cache.use_query_cache">true</prop> <prop key="hibernate.cache.region.factory_class"> org.hibernate.cache.ehcache.SingletonEhCacheRegionFactory</prop> <prop key="hibernate.generate_statistics">true</prop> <prop key="hibernate.cache.use_structured_entries">true</prop> <prop key="hibernate.cache.use_query_cache">true</prop> <prop key="hibernate.transaction.factory_class"> org.hibernate.engine.transaction.internal.jta.JtaTransactionFactory</prop> <prop key="hibernate.transaction.jta.platform"> org.hibernate.service.jta.platform.internal.JBossStandAloneJtaPlatform</prop> </props> </property> </bean> The testcase I'm running: @Test public void testGetParentFromCache() { for (int i = 0; i <3 ; i++ ) { getEntity(); } } private void getEntity() { Session sess = sessionFactory.openSession() sess.setCacheMode(CacheMode.NORMAL); Transaction t = sess.beginTransaction(); Parent p = (Parent) s.get(Parent.class, 123); Assert.assertNotNull(p); Assert.assertNotNull(p.getChildren().size()); t.commit(); sess.flush(); sess.clear(); sess.close(); } In the logging I can see that the first time 2 queries are executed getting the parent and getting the children. Furthermore the logging shows that the child entities as well as the collection are stored in the 2nd level cache. However when reading the collection a query is executed to fetch the children on second and third attempt.

    Read the article

  • Unable to Calculate Position within Owner-Draw Text

    - by Jonathan Wood
    I'm trying to use Visual Studio 2012 to create a Windows Forms application that can place the caret at the current position within a owner-drawn string. However, I've been unable to find a way to accurately calculate that position. I've done this successfully before in C++. I've now tried numerous methods in C#. Originally, I tried using .NET classes to determine the correct position, but then I tried accessing the Windows API directly. In some cases, I came close, but after some time I still cannot place the caret accurately. I've created a small test program and posted key parts below. I've also posted the entire project here. The exact font used is not important to me; however, my application assumes a mono-spaced font. Any help is appreciated. Form1.cs This is my main form. public partial class Form1 : Form { private string TestString; private int AveCharWidth; private int Position; public Form1() { InitializeComponent(); TestString = "123456789012345678901234567890123456789012345678901234567890"; AveCharWidth = GetFontWidth(); Position = 0; } private void Form1_Load(object sender, EventArgs e) { Font = new Font(FontFamily.GenericMonospace, 12, FontStyle.Regular, GraphicsUnit.Pixel); } protected override void OnGotFocus(EventArgs e) { Windows.CreateCaret(Handle, (IntPtr)0, 2, (int)Font.Height); Windows.ShowCaret(Handle); UpdateCaretPosition(); base.OnGotFocus(e); } protected void UpdateCaretPosition() { Windows.SetCaretPos(Padding.Left + (Position * AveCharWidth), Padding.Top); } protected override void OnLostFocus(EventArgs e) { Windows.HideCaret(Handle); Windows.DestroyCaret(); base.OnLostFocus(e); } protected override void OnPaint(PaintEventArgs e) { e.Graphics.DrawString(TestString, Font, SystemBrushes.WindowText, new PointF(Padding.Left, Padding.Top)); } protected override bool IsInputKey(Keys keyData) { switch (keyData) { case Keys.Right: case Keys.Left: return true; } return base.IsInputKey(keyData); } protected override void OnKeyDown(KeyEventArgs e) { switch (e.KeyCode) { case Keys.Left: Position = Math.Max(Position - 1, 0); UpdateCaretPosition(); break; case Keys.Right: Position = Math.Min(Position + 1, TestString.Length); UpdateCaretPosition(); break; } base.OnKeyDown(e); } protected int GetFontWidth() { int AverageCharWidth = 0; using (var graphics = this.CreateGraphics()) { try { Windows.TEXTMETRIC tm; var hdc = graphics.GetHdc(); IntPtr hFont = this.Font.ToHfont(); IntPtr hOldFont = Windows.SelectObject(hdc, hFont); var a = Windows.GetTextMetrics(hdc, out tm); var b = Windows.SelectObject(hdc, hOldFont); var c = Windows.DeleteObject(hFont); AverageCharWidth = tm.tmAveCharWidth; } catch { } finally { graphics.ReleaseHdc(); } } return AverageCharWidth; } } Windows.cs Here are my Windows API declarations. public static class Windows { [Serializable, StructLayout(LayoutKind.Sequential, CharSet = CharSet.Auto)] public struct TEXTMETRIC { public int tmHeight; public int tmAscent; public int tmDescent; public int tmInternalLeading; public int tmExternalLeading; public int tmAveCharWidth; public int tmMaxCharWidth; public int tmWeight; public int tmOverhang; public int tmDigitizedAspectX; public int tmDigitizedAspectY; public short tmFirstChar; public short tmLastChar; public short tmDefaultChar; public short tmBreakChar; public byte tmItalic; public byte tmUnderlined; public byte tmStruckOut; public byte tmPitchAndFamily; public byte tmCharSet; } [DllImport("user32.dll")] public static extern bool CreateCaret(IntPtr hWnd, IntPtr hBitmap, int nWidth, int nHeight); [DllImport("User32.dll")] public static extern bool SetCaretPos(int x, int y); [DllImport("User32.dll")] public static extern bool DestroyCaret(); [DllImport("User32.dll")] public static extern bool ShowCaret(IntPtr hWnd); [DllImport("User32.dll")] public static extern bool HideCaret(IntPtr hWnd); [DllImport("gdi32.dll", CharSet = CharSet.Auto)] public static extern bool GetTextMetrics(IntPtr hdc, out TEXTMETRIC lptm); [DllImport("gdi32.dll")] public static extern IntPtr SelectObject(IntPtr hdc, IntPtr hgdiobj); [DllImport("GDI32.dll")] public static extern bool DeleteObject(IntPtr hObject); }

    Read the article

  • having trouble with jpa, looks to be reading from cache when told to ignore

    - by jeff
    i'm using jpa and eclipselink. it says version 2.0.2.v20100323-r6872 for eclipselink. it's an SE application, small. local persistence unit. i am using a postgres database. i have some code the wakes up once per second and does a jpa query on a table. it's polling to see whether some fields on a given record change. but when i update a field in sql, it keeps showing the same value each time i query it. and this is after waiting, creating a new entitymanager, and giving a query hint. when i query the table from sql or python, i can see the changed field. but from within my jpa query, once i've read the value, it never changes in later executions of the query -- even though the entity manager has been recreated. i've tried telling it to not look in the cache. but it seems to ignore that. very puzzling, can you help please? here is the method. i call this once every 3 seconds. the tgt.getTrlDlrs() println shows me i get the same value for this field on every call of the method ("value won't change"). even if i change the field in the database. when i query the record from outside java, i see the change right away. also, if i stop the java program and restart it, i see the new value printed out immediately: public void exitChk(EntityManagerFactory emf, Main_1 mn){ // this will be a list of the trade close targets that are active List<Pairs01Tradeclosetgt> mn_res = new ArrayList<Pairs01Tradeclosetgt>(); //this is a list of the place number in the array list above //of positions to close ArrayList<Integer> idxsToCl = new ArrayList<Integer>(); EntityManager em = emf.createEntityManager(); em.getTransaction().begin(); em.clear(); String qryTxt = "SELECT p FROM Pairs01Tradeclosetgt p WHERE p.isClosed = :isClosed AND p.isFilled = :isFilled"; Query qMn = em.createQuery(qryTxt); qMn.setHint(QueryHints.CACHE_USAGE, CacheUsage.DoNotCheckCache); qMn.setParameter("isClosed", false); qMn.setParameter("isFilled", true); mn_res = (List<Pairs01Tradeclosetgt>) qMn.getResultList(); // are there any trade close targets we need to check for closing? if (mn_res!=null){ //if yes, see whether they've hit their target for (int i=0;i<mn_res.size();i++){ Pairs01Tradeclosetgt tgt = new Pairs01Tradeclosetgt(); tgt = mn_res.get(i); System.out.println("value won't change:" + tgt.getTrlDlrs()); here is my entity class (partial): @Entity @Table(name = "pairs01_tradeclosetgt", catalog = "blah", schema = "public") @NamedQueries({ @NamedQuery(name = "Pairs01Tradeclosetgt.findAll", query = "SELECT p FROM Pairs01Tradeclosetgt p"), @NamedQuery(name = "Pairs01Tradeclosetgt.findByClseRatio", query = "SELECT p FROM Pairs01Tradeclosetgt p WHERE p.clseRatio = :clseRatio")}) public class Pairs01Tradeclosetgt implements Serializable { private static final long serialVersionUID = 1L; @Id @Basic(optional = false) @Column(name = "id") @SequenceGenerator(name="pairs01_tradeclosetgt_id_seq", allocationSize=1) @GeneratedValue(strategy=GenerationType.SEQUENCE, generator = "pairs01_tradeclosetgt_id_seq") private Integer id; and my persitence unit: <?xml version="1.0" encoding="UTF-8"?> <persistence version="1.0" xmlns="http://java.sun.com/xml/ns/persistence" xmlns:xsi="http://www.w3.org/2001/XMLSchema-instance" xsi:schemaLocation="http://java.sun.com/xml/ns/persistence http://java.sun.com/xml/ns/persistence/persistence_1_0.xsd"> <persistence-unit name="testpu" transaction-type="RESOURCE_LOCAL"> <provider>org.eclipse.persistence.jpa.PersistenceProvider</provider> <class>mourv02.Pairs01Quotes</class> <class>mourv02.Pairs01Pair</class> <class>mourv02.Pairs01Trderrs</class> <class>mourv02.Pairs01Tradereq</class> <class>mourv02.Pairs01Tradeclosetgt</class> <class>mourv02.Pairs01Orderstatus</class> <properties> <property name="javax.persistence.jdbc.url" value="jdbc:postgresql://192.168.1.80:5432/blah"/> <property name="javax.persistence.jdbc.password" value="secret"/> <property name="javax.persistence.jdbc.driver" value="org.postgresql.Driver"/> <property name="javax.persistence.jdbc.user" value="noone"/> </properties> </persistence-unit> </persistence>

    Read the article

  • How can I map "insert='false' update='false'" on a composite-id key-property which is also used in a one-to-many FK?

    - by Gweebz
    I am working on a legacy code base with an existing DB schema. The existing code uses SQL and PL/SQL to execute queries on the DB. We have been tasked with making a small part of the project database-engine agnostic (at first, change everything eventually). We have chosen to use Hibernate 3.3.2.GA and "*.hbm.xml" mapping files (as opposed to annotations). Unfortunately, it is not feasible to change the existing schema because we cannot regress any legacy features. The problem I am encountering is when I am trying to map a uni-directional, one-to-many relationship where the FK is also part of a composite PK. Here are the classes and mapping file... CompanyEntity.java public class CompanyEntity { private Integer id; private Set<CompanyNameEntity> names; ... } CompanyNameEntity.java public class CompanyNameEntity implements Serializable { private Integer id; private String languageId; private String name; ... } CompanyNameEntity.hbm.xml <?xml version="1.0"?> <!DOCTYPE hibernate-mapping PUBLIC "-//Hibernate/Hibernate Mapping DTD 3.0//EN" "http://www.jboss.org/dtd/hibernate/hibernate-mapping-3.0.dtd"> <hibernate-mapping package="com.example"> <class name="com.example.CompanyEntity" table="COMPANY"> <id name="id" column="COMPANY_ID"/> <set name="names" table="COMPANY_NAME" cascade="all-delete-orphan" fetch="join" batch-size="1" lazy="false"> <key column="COMPANY_ID"/> <one-to-many entity-name="vendorName"/> </set> </class> <class entity-name="companyName" name="com.example.CompanyNameEntity" table="COMPANY_NAME"> <composite-id> <key-property name="id" column="COMPANY_ID"/> <key-property name="languageId" column="LANGUAGE_ID"/> </composite-id> <property name="name" column="NAME" length="255"/> </class> </hibernate-mapping> This code works just fine for SELECT and INSERT of a Company with names. I encountered a problem when I tried to update and existing record. I received a BatchUpdateException and after looking through the SQL logs I saw Hibernate was trying to do something stupid... update COMPANY_NAME set COMPANY_ID=null where COMPANY_ID=? Hibernate was trying to dis-associate child records before updating them. The problem is that this field is part of the PK and not-nullable. I found the quick solution to make Hibernate not do this is to add "not-null='true'" to the "key" element in the parent mapping. SO now may mapping looks like this... CompanyNameEntity.hbm.xml <?xml version="1.0"?> <!DOCTYPE hibernate-mapping PUBLIC "-//Hibernate/Hibernate Mapping DTD 3.0//EN" "http://www.jboss.org/dtd/hibernate/hibernate-mapping-3.0.dtd"> <hibernate-mapping package="com.example"> <class name="com.example.CompanyEntity" table="COMPANY"> <id name="id" column="COMPANY_ID"/> <set name="names" table="COMPANY_NAME" cascade="all-delete-orphan" fetch="join" batch-size="1" lazy="false"> <key column="COMPANY_ID" not-null="true"/> <one-to-many entity-name="vendorName"/> </set> </class> <class entity-name="companyName" name="com.example.CompanyNameEntity" table="COMPANY_NAME"> <composite-id> <key-property name="id" column="COMPANY_ID"/> <key-property name="languageId" column="LANGUAGE_ID"/> </composite-id> <property name="name" column="NAME" length="255"/> </class> </hibernate-mapping> This mapping gives the exception... org.hibernate.MappingException: Repeated column in mapping for entity: companyName column: COMPANY_ID (should be mapped with insert="false" update="false") My problem now is that I have tryed to add these attributes to the key-property element but that is not supported by the DTD. I have also tryed changing it to a key-many-to-one element but that didn't work either. So... How can I map "insert='false' update='false'" on a composite-id key-property which is also used in a one-to-many FK?

    Read the article

  • Java programming accessing object variables

    - by Haxed
    Helo, there are 3 files, CustomerClient.java, CustomerServer.java and Customer.java PROBLEM: In the CustomerServer.java file, i get an error when I compile the CustomerServer.java at line : System.out.println(a[k].getName()); ERROR: init: deps-jar: Compiling 1 source file to C:\Documents and Settings\TLNA\My Documents\NetBeansProjects\Server\build\classes C:\Documents and Settings\TLNA\My Documents\NetBeansProjects\Server\src\CustomerServer.java:44: cannot find symbol symbol : method getName() location: class Customer System.out.println(a[k].getName()); 1 error BUILD FAILED (total time: 0 seconds) CustomerClient.java import java.io.*; import java.net.*; import java.awt.*; import java.awt.event.*; import javax.swing.*; import javax.swing.border.*; public class CustomerClient extends JApplet { private JTextField jtfName = new JTextField(32); private JTextField jtfSeatNo = new JTextField(32); // Button for sending a student to the server private JButton jbtRegister = new JButton("Register to the Server"); // Indicate if it runs as application private boolean isStandAlone = false; // Host name or ip String host = "localhost"; public void init() { JPanel p1 = new JPanel(); p1.setLayout(new GridLayout(2, 1)); p1.add(new JLabel("Name")); p1.add(jtfName); p1.add(new JLabel("Seat No.")); p1.add(jtfSeatNo); add(p1, BorderLayout.CENTER); add(jbtRegister, BorderLayout.SOUTH); // Register listener jbtRegister.addActionListener(new ButtonListener()); // Find the IP address of the Web server if (!isStandAlone) { host = getCodeBase().getHost(); } } /** Handle button action */ private class ButtonListener implements ActionListener { public void actionPerformed(ActionEvent e) { try { // Establish connection with the server Socket socket = new Socket(host, 8000); // Create an output stream to the server ObjectOutputStream toServer = new ObjectOutputStream(socket.getOutputStream()); // Get text field String name = jtfName.getText().trim(); String seatNo = jtfSeatNo.getText().trim(); // Create a Student object and send to the server Customer s = new Customer(name, seatNo); toServer.writeObject(s); } catch (IOException ex) { System.err.println(ex); } } } /** Run the applet as an application */ public static void main(String[] args) { // Create a frame JFrame frame = new JFrame("Register Student Client"); // Create an instance of the applet CustomerClient applet = new CustomerClient(); applet.isStandAlone = true; // Get host if (args.length == 1) { applet.host = args[0]; // Add the applet instance to the frame } frame.add(applet, BorderLayout.CENTER); // Invoke init() and start() applet.init(); applet.start(); // Display the frame frame.pack(); frame.setVisible(true); } } CustomerServer.java import java.io.*; import java.net.*; public class CustomerServer { private String name; private int i; private ObjectOutputStream outputToFile; private ObjectInputStream inputFromClient; public static void main(String[] args) { new CustomerServer(); } public CustomerServer() { Customer[] a = new Customer[30]; try { // Create a server socket ServerSocket serverSocket = new ServerSocket(8000); System.out.println("Server started "); // Create an object ouput stream outputToFile = new ObjectOutputStream( new FileOutputStream("student.dat", true)); while (true) { // Listen for a new connection request Socket socket = serverSocket.accept(); // Create an input stream from the socket inputFromClient = new ObjectInputStream(socket.getInputStream()); // Read from input //Object object = inputFromClient.readObject(); for (int k = 0; k <= 2; k++) { if (a[k] == null) { a[k] = (Customer) inputFromClient.readObject(); // Write to the file outputToFile.writeObject(a[k]); //System.out.println("A new student object is stored"); System.out.println(a[k].getName()); break; } if (k == 2) { //fully booked outputToFile.writeObject("All seats are booked"); break; } } } } catch (ClassNotFoundException ex) { ex.printStackTrace(); } catch (IOException ex) { ex.printStackTrace(); } finally { try { inputFromClient.close(); outputToFile.close(); } catch (Exception ex) { ex.printStackTrace(); } } } } Customer.java public class Customer implements java.io.Serializable { private String name; private String seatno; public Customer(String name, String seatno) { this.name = name; this.seatno = seatno; } public String getName() { return name; } public String getSeatNo() { return seatno; } }

    Read the article

  • Creating a dynamic, extensible C# Expando Object

    - by Rick Strahl
    I love dynamic functionality in a strongly typed language because it offers us the best of both worlds. In C# (or any of the main .NET languages) we now have the dynamic type that provides a host of dynamic features for the static C# language. One place where I've found dynamic to be incredibly useful is in building extensible types or types that expose traditionally non-object data (like dictionaries) in easier to use and more readable syntax. I wrote about a couple of these for accessing old school ADO.NET DataRows and DataReaders more easily for example. These classes are dynamic wrappers that provide easier syntax and auto-type conversions which greatly simplifies code clutter and increases clarity in existing code. ExpandoObject in .NET 4.0 Another great use case for dynamic objects is the ability to create extensible objects - objects that start out with a set of static members and then can add additional properties and even methods dynamically. The .NET 4.0 framework actually includes an ExpandoObject class which provides a very dynamic object that allows you to add properties and methods on the fly and then access them again. For example with ExpandoObject you can do stuff like this:dynamic expand = new ExpandoObject(); expand.Name = "Rick"; expand.HelloWorld = (Func<string, string>) ((string name) => { return "Hello " + name; }); Console.WriteLine(expand.Name); Console.WriteLine(expand.HelloWorld("Dufus")); Internally ExpandoObject uses a Dictionary like structure and interface to store properties and methods and then allows you to add and access properties and methods easily. As cool as ExpandoObject is it has a few shortcomings too: It's a sealed type so you can't use it as a base class It only works off 'properties' in the internal Dictionary - you can't expose existing type data It doesn't serialize to XML or with DataContractSerializer/DataContractJsonSerializer Expando - A truly extensible Object ExpandoObject is nice if you just need a dynamic container for a dictionary like structure. However, if you want to build an extensible object that starts out with a set of strongly typed properties and then allows you to extend it, ExpandoObject does not work because it's a sealed class that can't be inherited. I started thinking about this very scenario for one of my applications I'm building for a customer. In this system we are connecting to various different user stores. Each user store has the same basic requirements for username, password, name etc. But then each store also has a number of extended properties that is available to each application. In the real world scenario the data is loaded from the database in a data reader and the known properties are assigned from the known fields in the database. All unknown fields are then 'added' to the expando object dynamically. In the past I've done this very thing with a separate property - Properties - just like I do for this class. But the property and dictionary syntax is not ideal and tedious to work with. I started thinking about how to represent these extra property structures. One way certainly would be to add a Dictionary, or an ExpandoObject to hold all those extra properties. But wouldn't it be nice if the application could actually extend an existing object that looks something like this as you can with the Expando object:public class User : Westwind.Utilities.Dynamic.Expando { public string Email { get; set; } public string Password { get; set; } public string Name { get; set; } public bool Active { get; set; } public DateTime? ExpiresOn { get; set; } } and then simply start extending the properties of this object dynamically? Using the Expando object I describe later you can now do the following:[TestMethod] public void UserExampleTest() { var user = new User(); // Set strongly typed properties user.Email = "[email protected]"; user.Password = "nonya123"; user.Name = "Rickochet"; user.Active = true; // Now add dynamic properties dynamic duser = user; duser.Entered = DateTime.Now; duser.Accesses = 1; // you can also add dynamic props via indexer user["NickName"] = "AntiSocialX"; duser["WebSite"] = "http://www.west-wind.com/weblog"; // Access strong type through dynamic ref Assert.AreEqual(user.Name,duser.Name); // Access strong type through indexer Assert.AreEqual(user.Password,user["Password"]); // access dyanmically added value through indexer Assert.AreEqual(duser.Entered,user["Entered"]); // access index added value through dynamic Assert.AreEqual(user["NickName"],duser.NickName); // loop through all properties dynamic AND strong type properties (true) foreach (var prop in user.GetProperties(true)) { object val = prop.Value; if (val == null) val = "null"; Console.WriteLine(prop.Key + ": " + val.ToString()); } } As you can see this code somewhat blurs the line between a static and dynamic type. You start with a strongly typed object that has a fixed set of properties. You can then cast the object to dynamic (as I discussed in my last post) and add additional properties to the object. You can also use an indexer to add dynamic properties to the object. To access the strongly typed properties you can use either the strongly typed instance, the indexer or the dynamic cast of the object. Personally I think it's kinda cool to have an easy way to access strongly typed properties by string which can make some data scenarios much easier. To access the 'dynamically added' properties you can use either the indexer on the strongly typed object, or property syntax on the dynamic cast. Using the dynamic type allows all three modes to work on both strongly typed and dynamic properties. Finally you can iterate over all properties, both dynamic and strongly typed if you chose. Lots of flexibility. Note also that by default the Expando object works against the (this) instance meaning it extends the current object. You can also pass in a separate instance to the constructor in which case that object will be used to iterate over to find properties rather than this. Using this approach provides some really interesting functionality when use the dynamic type. To use this we have to add an explicit constructor to the Expando subclass:public class User : Westwind.Utilities.Dynamic.Expando { public string Email { get; set; } public string Password { get; set; } public string Name { get; set; } public bool Active { get; set; } public DateTime? ExpiresOn { get; set; } public User() : base() { } // only required if you want to mix in seperate instance public User(object instance) : base(instance) { } } to allow the instance to be passed. When you do you can now do:[TestMethod] public void ExpandoMixinTest() { // have Expando work on Addresses var user = new User( new Address() ); // cast to dynamicAccessToPropertyTest dynamic duser = user; // Set strongly typed properties duser.Email = "[email protected]"; user.Password = "nonya123"; // Set properties on address object duser.Address = "32 Kaiea"; //duser.Phone = "808-123-2131"; // set dynamic properties duser.NonExistantProperty = "This works too"; // shows default value Address.Phone value Console.WriteLine(duser.Phone); } Using the dynamic cast in this case allows you to access *three* different 'objects': The strong type properties, the dynamically added properties in the dictionary and the properties of the instance passed in! Effectively this gives you a way to simulate multiple inheritance (which is scary - so be very careful with this, but you can do it). How Expando works Behind the scenes Expando is a DynamicObject subclass as I discussed in my last post. By implementing a few of DynamicObject's methods you can basically create a type that can trap 'property missing' and 'method missing' operations. When you access a non-existant property a known method is fired that our code can intercept and provide a value for. Internally Expando uses a custom dictionary implementation to hold the dynamic properties you might add to your expandable object. Let's look at code first. The code for the Expando type is straight forward and given what it provides relatively short. Here it is.using System; using System.Collections.Generic; using System.Linq; using System.Dynamic; using System.Reflection; namespace Westwind.Utilities.Dynamic { /// <summary> /// Class that provides extensible properties and methods. This /// dynamic object stores 'extra' properties in a dictionary or /// checks the actual properties of the instance. /// /// This means you can subclass this expando and retrieve either /// native properties or properties from values in the dictionary. /// /// This type allows you three ways to access its properties: /// /// Directly: any explicitly declared properties are accessible /// Dynamic: dynamic cast allows access to dictionary and native properties/methods /// Dictionary: Any of the extended properties are accessible via IDictionary interface /// </summary> [Serializable] public class Expando : DynamicObject, IDynamicMetaObjectProvider { /// <summary> /// Instance of object passed in /// </summary> object Instance; /// <summary> /// Cached type of the instance /// </summary> Type InstanceType; PropertyInfo[] InstancePropertyInfo { get { if (_InstancePropertyInfo == null && Instance != null) _InstancePropertyInfo = Instance.GetType().GetProperties(BindingFlags.Instance | BindingFlags.Public | BindingFlags.DeclaredOnly); return _InstancePropertyInfo; } } PropertyInfo[] _InstancePropertyInfo; /// <summary> /// String Dictionary that contains the extra dynamic values /// stored on this object/instance /// </summary> /// <remarks>Using PropertyBag to support XML Serialization of the dictionary</remarks> public PropertyBag Properties = new PropertyBag(); //public Dictionary<string,object> Properties = new Dictionary<string, object>(); /// <summary> /// This constructor just works off the internal dictionary and any /// public properties of this object. /// /// Note you can subclass Expando. /// </summary> public Expando() { Initialize(this); } /// <summary> /// Allows passing in an existing instance variable to 'extend'. /// </summary> /// <remarks> /// You can pass in null here if you don't want to /// check native properties and only check the Dictionary! /// </remarks> /// <param name="instance"></param> public Expando(object instance) { Initialize(instance); } protected virtual void Initialize(object instance) { Instance = instance; if (instance != null) InstanceType = instance.GetType(); } /// <summary> /// Try to retrieve a member by name first from instance properties /// followed by the collection entries. /// </summary> /// <param name="binder"></param> /// <param name="result"></param> /// <returns></returns> public override bool TryGetMember(GetMemberBinder binder, out object result) { result = null; // first check the Properties collection for member if (Properties.Keys.Contains(binder.Name)) { result = Properties[binder.Name]; return true; } // Next check for Public properties via Reflection if (Instance != null) { try { return GetProperty(Instance, binder.Name, out result); } catch { } } // failed to retrieve a property result = null; return false; } /// <summary> /// Property setter implementation tries to retrieve value from instance /// first then into this object /// </summary> /// <param name="binder"></param> /// <param name="value"></param> /// <returns></returns> public override bool TrySetMember(SetMemberBinder binder, object value) { // first check to see if there's a native property to set if (Instance != null) { try { bool result = SetProperty(Instance, binder.Name, value); if (result) return true; } catch { } } // no match - set or add to dictionary Properties[binder.Name] = value; return true; } /// <summary> /// Dynamic invocation method. Currently allows only for Reflection based /// operation (no ability to add methods dynamically). /// </summary> /// <param name="binder"></param> /// <param name="args"></param> /// <param name="result"></param> /// <returns></returns> public override bool TryInvokeMember(InvokeMemberBinder binder, object[] args, out object result) { if (Instance != null) { try { // check instance passed in for methods to invoke if (InvokeMethod(Instance, binder.Name, args, out result)) return true; } catch { } } result = null; return false; } /// <summary> /// Reflection Helper method to retrieve a property /// </summary> /// <param name="instance"></param> /// <param name="name"></param> /// <param name="result"></param> /// <returns></returns> protected bool GetProperty(object instance, string name, out object result) { if (instance == null) instance = this; var miArray = InstanceType.GetMember(name, BindingFlags.Public | BindingFlags.GetProperty | BindingFlags.Instance); if (miArray != null && miArray.Length > 0) { var mi = miArray[0]; if (mi.MemberType == MemberTypes.Property) { result = ((PropertyInfo)mi).GetValue(instance,null); return true; } } result = null; return false; } /// <summary> /// Reflection helper method to set a property value /// </summary> /// <param name="instance"></param> /// <param name="name"></param> /// <param name="value"></param> /// <returns></returns> protected bool SetProperty(object instance, string name, object value) { if (instance == null) instance = this; var miArray = InstanceType.GetMember(name, BindingFlags.Public | BindingFlags.SetProperty | BindingFlags.Instance); if (miArray != null && miArray.Length > 0) { var mi = miArray[0]; if (mi.MemberType == MemberTypes.Property) { ((PropertyInfo)mi).SetValue(Instance, value, null); return true; } } return false; } /// <summary> /// Reflection helper method to invoke a method /// </summary> /// <param name="instance"></param> /// <param name="name"></param> /// <param name="args"></param> /// <param name="result"></param> /// <returns></returns> protected bool InvokeMethod(object instance, string name, object[] args, out object result) { if (instance == null) instance = this; // Look at the instanceType var miArray = InstanceType.GetMember(name, BindingFlags.InvokeMethod | BindingFlags.Public | BindingFlags.Instance); if (miArray != null && miArray.Length > 0) { var mi = miArray[0] as MethodInfo; result = mi.Invoke(Instance, args); return true; } result = null; return false; } /// <summary> /// Convenience method that provides a string Indexer /// to the Properties collection AND the strongly typed /// properties of the object by name. /// /// // dynamic /// exp["Address"] = "112 nowhere lane"; /// // strong /// var name = exp["StronglyTypedProperty"] as string; /// </summary> /// <remarks> /// The getter checks the Properties dictionary first /// then looks in PropertyInfo for properties. /// The setter checks the instance properties before /// checking the Properties dictionary. /// </remarks> /// <param name="key"></param> /// /// <returns></returns> public object this[string key] { get { try { // try to get from properties collection first return Properties[key]; } catch (KeyNotFoundException ex) { // try reflection on instanceType object result = null; if (GetProperty(Instance, key, out result)) return result; // nope doesn't exist throw; } } set { if (Properties.ContainsKey(key)) { Properties[key] = value; return; } // check instance for existance of type first var miArray = InstanceType.GetMember(key, BindingFlags.Public | BindingFlags.GetProperty); if (miArray != null && miArray.Length > 0) SetProperty(Instance, key, value); else Properties[key] = value; } } /// <summary> /// Returns and the properties of /// </summary> /// <param name="includeProperties"></param> /// <returns></returns> public IEnumerable<KeyValuePair<string,object>> GetProperties(bool includeInstanceProperties = false) { if (includeInstanceProperties && Instance != null) { foreach (var prop in this.InstancePropertyInfo) yield return new KeyValuePair<string, object>(prop.Name, prop.GetValue(Instance, null)); } foreach (var key in this.Properties.Keys) yield return new KeyValuePair<string, object>(key, this.Properties[key]); } /// <summary> /// Checks whether a property exists in the Property collection /// or as a property on the instance /// </summary> /// <param name="item"></param> /// <returns></returns> public bool Contains(KeyValuePair<string, object> item, bool includeInstanceProperties = false) { bool res = Properties.ContainsKey(item.Key); if (res) return true; if (includeInstanceProperties && Instance != null) { foreach (var prop in this.InstancePropertyInfo) { if (prop.Name == item.Key) return true; } } return false; } } } Although the Expando class supports an indexer, it doesn't actually implement IDictionary or even IEnumerable. It only provides the indexer and Contains() and GetProperties() methods, that work against the Properties dictionary AND the internal instance. The reason for not implementing IDictionary is that a) it doesn't add much value since you can access the Properties dictionary directly and that b) I wanted to keep the interface to class very lean so that it can serve as an entity type if desired. Implementing these IDictionary (or even IEnumerable) causes LINQ extension methods to pop up on the type which obscures the property interface and would only confuse the purpose of the type. IDictionary and IEnumerable are also problematic for XML and JSON Serialization - the XML Serializer doesn't serialize IDictionary<string,object>, nor does the DataContractSerializer. The JavaScriptSerializer does serialize, but it treats the entire object like a dictionary and doesn't serialize the strongly typed properties of the type, only the dictionary values which is also not desirable. Hence the decision to stick with only implementing the indexer to support the user["CustomProperty"] functionality and leaving iteration functions to the publicly exposed Properties dictionary. Note that the Dictionary used here is a custom PropertyBag class I created to allow for serialization to work. One important aspect for my apps is that whatever custom properties get added they have to be accessible to AJAX clients since the particular app I'm working on is a SIngle Page Web app where most of the Web access is through JSON AJAX calls. PropertyBag can serialize to XML and one way serialize to JSON using the JavaScript serializer (not the DCS serializers though). The key components that make Expando work in this code are the Properties Dictionary and the TryGetMember() and TrySetMember() methods. The Properties collection is public so if you choose you can explicitly access the collection to get better performance or to manipulate the members in internal code (like loading up dynamic values form a database). Notice that TryGetMember() and TrySetMember() both work against the dictionary AND the internal instance to retrieve and set properties. This means that user["Name"] works against native properties of the object as does user["Name"] = "RogaDugDog". What's your Use Case? This is still an early prototype but I've plugged it into one of my customer's applications and so far it's working very well. The key features for me were the ability to easily extend the type with values coming from a database and exposing those values in a nice and easy to use manner. I'm also finding that using this type of object for ViewModels works very well to add custom properties to view models. I suspect there will be lots of uses for this - I've been using the extra dictionary approach to extensibility for years - using a dynamic type to make the syntax cleaner is just a bonus here. What can you think of to use this for? Resources Source Code and Tests (GitHub) Also integrated in Westwind.Utilities of the West Wind Web Toolkit West Wind Utilities NuGet© Rick Strahl, West Wind Technologies, 2005-2012Posted in CSharp  .NET  Dynamic Types   Tweet !function(d,s,id){var js,fjs=d.getElementsByTagName(s)[0];if(!d.getElementById(id)){js=d.createElement(s);js.id=id;js.src="//platform.twitter.com/widgets.js";fjs.parentNode.insertBefore(js,fjs);}}(document,"script","twitter-wjs"); (function() { var po = document.createElement('script'); po.type = 'text/javascript'; po.async = true; po.src = 'https://apis.google.com/js/plusone.js'; var s = document.getElementsByTagName('script')[0]; s.parentNode.insertBefore(po, s); })();

    Read the article

  • Using jQuery to POST Form Data to an ASP.NET ASMX AJAX Web Service

    - by Rick Strahl
    The other day I got a question about how to call an ASP.NET ASMX Web Service or PageMethods with the POST data from a Web Form (or any HTML form for that matter). The idea is that you should be able to call an endpoint URL, send it regular urlencoded POST data and then use Request.Form[] to retrieve the posted data as needed. My first reaction was that you can’t do it, because ASP.NET ASMX AJAX services (as well as Page Methods and WCF REST AJAX Services) require that the content POSTed to the server is posted as JSON and sent with an application/json or application/x-javascript content type. IOW, you can’t directly call an ASP.NET AJAX service with regular urlencoded data. Note that there are other ways to accomplish this. You can use ASP.NET MVC and a custom route, an HTTP Handler or separate ASPX page, or even a WCF REST service that’s configured to use non-JSON inputs. However if you want to use an ASP.NET AJAX service (or Page Methods) with a little bit of setup work it’s actually quite easy to capture all the form variables on the client and ship them up to the server. The basic steps needed to make this happen are: Capture form variables into an array on the client with jQuery’s .serializeArray() function Use $.ajax() or my ServiceProxy class to make an AJAX call to the server to send this array On the server create a custom type that matches the .serializeArray() name/value structure Create extension methods on NameValue[] to easily extract form variables Create a [WebMethod] that accepts this name/value type as an array (NameValue[]) This seems like a lot of work but realize that steps 3 and 4 are a one time setup step that can be reused in your entire site or multiple applications. Let’s look at a short example that looks like this as a base form of fields to ship to the server: The HTML for this form looks something like this: <div id="divMessage" class="errordisplay" style="display: none"> </div> <div> <div class="label">Name:</div> <div><asp:TextBox runat="server" ID="txtName" /></div> </div> <div> <div class="label">Company:</div> <div><asp:TextBox runat="server" ID="txtCompany"/></div> </div> <div> <div class="label" ></div> <div> <asp:DropDownList runat="server" ID="lstAttending"> <asp:ListItem Text="Attending" Value="Attending"/> <asp:ListItem Text="Not Attending" Value="NotAttending" /> <asp:ListItem Text="Maybe Attending" Value="MaybeAttending" /> <asp:ListItem Text="Not Sure Yet" Value="NotSureYet" /> </asp:DropDownList> </div> </div> <div> <div class="label">Special Needs:<br /> <small>(check all that apply)</small></div> <div> <asp:ListBox runat="server" ID="lstSpecialNeeds" SelectionMode="Multiple"> <asp:ListItem Text="Vegitarian" Value="Vegitarian" /> <asp:ListItem Text="Vegan" Value="Vegan" /> <asp:ListItem Text="Kosher" Value="Kosher" /> <asp:ListItem Text="Special Access" Value="SpecialAccess" /> <asp:ListItem Text="No Binder" Value="NoBinder" /> </asp:ListBox> </div> </div> <div> <div class="label"></div> <div> <asp:CheckBox ID="chkAdditionalGuests" Text="Additional Guests" runat="server" /> </div> </div> <hr /> <input type="button" id="btnSubmit" value="Send Registration" /> The form includes a few different kinds of form fields including a multi-selection listbox to demonstrate retrieving multiple values. Setting up the Server Side [WebMethod] The [WebMethod] on the server we’re going to call is going to be very simple and just capture the content of these values and echo then back as a formatted HTML string. Obviously this is overly simplistic but it serves to demonstrate the simple point of capturing the POST data on the server in an AJAX callback. public class PageMethodsService : System.Web.Services.WebService { [WebMethod] public string SendRegistration(NameValue[] formVars) { StringBuilder sb = new StringBuilder(); sb.AppendFormat("Thank you {0}, <br/><br/>", HttpUtility.HtmlEncode(formVars.Form("txtName"))); sb.AppendLine("You've entered the following: <hr/>"); foreach (NameValue nv in formVars) { // strip out ASP.NET form vars like _ViewState/_EventValidation if (!nv.name.StartsWith("__")) { if (nv.name.StartsWith("txt") || nv.name.StartsWith("lst") || nv.name.StartsWith("chk")) sb.Append(nv.name.Substring(3)); else sb.Append(nv.name); sb.AppendLine(": " + HttpUtility.HtmlEncode(nv.value) + "<br/>"); } } sb.AppendLine("<hr/>"); string[] needs = formVars.FormMultiple("lstSpecialNeeds"); if (needs == null) sb.AppendLine("No Special Needs"); else { sb.AppendLine("Special Needs: <br/>"); foreach (string need in needs) { sb.AppendLine("&nbsp;&nbsp;" + need + "<br/>"); } } return sb.ToString(); } } The key feature of this method is that it receives a custom type called NameValue[] which is an array of NameValue objects that map the structure that the jQuery .serializeArray() function generates. There are two custom types involved in this: The actual NameValue type and a NameValueExtensions class that defines a couple of extension methods for the NameValue[] array type to allow for single (.Form()) and multiple (.FormMultiple()) value retrieval by name. The NameValue class is as simple as this and simply maps the structure of the array elements of .serializeArray(): public class NameValue { public string name { get; set; } public string value { get; set; } } The extension method class defines the .Form() and .FormMultiple() methods to allow easy retrieval of form variables from the returned array: /// <summary> /// Simple NameValue class that maps name and value /// properties that can be used with jQuery's /// $.serializeArray() function and JSON requests /// </summary> public static class NameValueExtensionMethods { /// <summary> /// Retrieves a single form variable from the list of /// form variables stored /// </summary> /// <param name="formVars"></param> /// <param name="name">formvar to retrieve</param> /// <returns>value or string.Empty if not found</returns> public static string Form(this NameValue[] formVars, string name) { var matches = formVars.Where(nv => nv.name.ToLower() == name.ToLower()).FirstOrDefault(); if (matches != null) return matches.value; return string.Empty; } /// <summary> /// Retrieves multiple selection form variables from the list of /// form variables stored. /// </summary> /// <param name="formVars"></param> /// <param name="name">The name of the form var to retrieve</param> /// <returns>values as string[] or null if no match is found</returns> public static string[] FormMultiple(this NameValue[] formVars, string name) { var matches = formVars.Where(nv => nv.name.ToLower() == name.ToLower()).Select(nv => nv.value).ToArray(); if (matches.Length == 0) return null; return matches; } } Using these extension methods it’s easy to retrieve individual values from the array: string name = formVars.Form("txtName"); or multiple values: string[] needs = formVars.FormMultiple("lstSpecialNeeds"); if (needs != null) { // do something with matches } Using these functions in the SendRegistration method it’s easy to retrieve a few form variables directly (txtName and the multiple selections of lstSpecialNeeds) or to iterate over the whole list of values. Of course this is an overly simple example – in typical app you’d probably want to validate the input data and save it to the database and then return some sort of confirmation or possibly an updated data list back to the client. Since this is a full AJAX service callback realize that you don’t have to return simple string values – you can return any of the supported result types (which are most serializable types) including complex hierarchical objects and arrays that make sense to your client code. POSTing Form Variables from the Client to the AJAX Service To call the AJAX service method on the client is straight forward and requires only use of little native jQuery plus JSON serialization functionality. To start add jQuery and the json2.js library to your page: <script src="Scripts/jquery.min.js" type="text/javascript"></script> <script src="Scripts/json2.js" type="text/javascript"></script> json2.js can be found here (be sure to remove the first line from the file): http://www.json.org/json2.js It’s required to handle JSON serialization for those browsers that don’t support it natively. With those script references in the document let’s hookup the button click handler and call the service: $(document).ready(function () { $("#btnSubmit").click(sendRegistration); }); function sendRegistration() { var arForm = $("#form1").serializeArray(); $.ajax({ url: "PageMethodsService.asmx/SendRegistration", type: "POST", contentType: "application/json", data: JSON.stringify({ formVars: arForm }), dataType: "json", success: function (result) { var jEl = $("#divMessage"); jEl.html(result.d).fadeIn(1000); setTimeout(function () { jEl.fadeOut(1000) }, 5000); }, error: function (xhr, status) { alert("An error occurred: " + status); } }); } The key feature in this code is the $("#form1").serializeArray();  call which serializes all the form fields of form1 into an array. Each form var is represented as an object with a name/value property. This array is then serialized into JSON with: JSON.stringify({ formVars: arForm }) The format for the parameter list in AJAX service calls is an object with one property for each parameter of the method. In this case its a single parameter called formVars and we’re assigning the array of form variables to it. The URL to call on the server is the name of the Service (or ASPX Page for Page Methods) plus the name of the method to call. On return the success callback receives the result from the AJAX callback which in this case is the formatted string which is simply assigned to an element in the form and displayed. Remember the result type is whatever the method returns – it doesn’t have to be a string. Note that ASP.NET AJAX and WCF REST return JSON data as a wrapped object so the result has a ‘d’ property that holds the actual response: jEl.html(result.d).fadeIn(1000); Slightly simpler: Using ServiceProxy.js If you want things slightly cleaner you can use the ServiceProxy.js class I’ve mentioned here before. The ServiceProxy class handles a few things for calling ASP.NET and WCF services more cleanly: Automatic JSON encoding Automatic fix up of ‘d’ wrapper property Automatic Date conversion on the client Simplified error handling Reusable and abstracted To add the service proxy add: <script src="Scripts/ServiceProxy.js" type="text/javascript"></script> and then change the code to this slightly simpler version: <script type="text/javascript"> proxy = new ServiceProxy("PageMethodsService.asmx/"); $(document).ready(function () { $("#btnSubmit").click(sendRegistration); }); function sendRegistration() { var arForm = $("#form1").serializeArray(); proxy.invoke("SendRegistration", { formVars: arForm }, function (result) { var jEl = $("#divMessage"); jEl.html(result).fadeIn(1000); setTimeout(function () { jEl.fadeOut(1000) }, 5000); }, function (error) { alert(error.message); } ); } The code is not very different but it makes the call as simple as specifying the method to call, the parameters to pass and the actions to take on success and error. No more remembering which content type and data types to use and manually serializing to JSON. This code also removes the “d” property processing in the response and provides more consistent error handling in that the call always returns an error object regardless of a server error or a communication error unlike the native $.ajax() call. Either approach works and both are pretty easy. The ServiceProxy really pays off if you use lots of service calls and especially if you need to deal with date values returned from the server  on the client. Summary Making Web Service calls and getting POST data to the server is not always the best option – ASP.NET and WCF AJAX services are meant to work with data in objects. However, in some situations it’s simply easier to POST all the captured form data to the server instead of mapping all properties from the input fields to some sort of message object first. For this approach the above POST mechanism is useful as it puts the parsing of the data on the server and leaves the client code lean and mean. It’s even easy to build a custom model binder on the server that can map the array values to properties on an object generically with some relatively simple Reflection code and without having to manually map form vars to properties and do string conversions. Keep in mind though that other approaches also abound. ASP.NET MVC makes it pretty easy to create custom routes to data and the built in model binder makes it very easy to deal with inbound form POST data in its original urlencoded format. The West Wind West Wind Web Toolkit also includes functionality for AJAX callbacks using plain POST values. All that’s needed is a Method parameter to query/form value to specify the method to be called on the server. After that the content type is completely optional and up to the consumer. It’d be nice if the ASP.NET AJAX Service and WCF AJAX Services weren’t so tightly bound to the content type so that you could more easily create open access service endpoints that can take advantage of urlencoded data that is everywhere in existing pages. It would make it much easier to create basic REST endpoints without complicated service configuration. Ah one can dream! In the meantime I hope this article has given you some ideas on how you can transfer POST data from the client to the server using JSON – it might be useful in other scenarios beyond ASP.NET AJAX services as well. Additional Resources ServiceProxy.js A small JavaScript library that wraps $.ajax() to call ASP.NET AJAX and WCF AJAX Services. Includes date parsing extensions to the JSON object, a global dataFilter for processing dates on all jQuery JSON requests, provides cleanup for the .NET wrapped message format and handles errors in a consistent fashion. Making jQuery Calls to WCF/ASMX with a ServiceProxy Client More information on calling ASMX and WCF AJAX services with jQuery and some more background on ServiceProxy.js. Note the implementation has slightly changed since the article was written. ww.jquery.js The West Wind West Wind Web Toolkit also includes ServiceProxy.js in the West Wind jQuery extension library. This version is slightly different and includes embedded json encoding/decoding based on json2.js.© Rick Strahl, West Wind Technologies, 2005-2010Posted in jQuery  ASP.NET  AJAX  

    Read the article

  • Hibernate Exception, what wrong ? [[Exception in thread "main" org.hibernate.InvalidMappingException

    - by user195970
    I use netbean 6.7.1 to write "hello world" witch hibernate, but I get some errors, plz help me, thank you very much. my exception init: deps-module-jar: deps-ear-jar: deps-jar: Copying 1 file to F:\Documents and Settings\My Dropbox\DropboxNetBeanProjects\loginspring\build\web\WEB-INF\classes compile-single: run-main: Oct 25, 2009 2:44:05 AM org.hibernate.cfg.Environment <clinit> INFO: Hibernate 3.2.5 Oct 25, 2009 2:44:05 AM org.hibernate.cfg.Environment <clinit> INFO: hibernate.properties not found Oct 25, 2009 2:44:05 AM org.hibernate.cfg.Environment buildBytecodeProvider INFO: Bytecode provider name : cglib Oct 25, 2009 2:44:05 AM org.hibernate.cfg.Environment <clinit> INFO: using JDK 1.4 java.sql.Timestamp handling Oct 25, 2009 2:44:05 AM org.hibernate.cfg.Configuration configure INFO: configuring from resource: /hibernate.cfg.xml Oct 25, 2009 2:44:05 AM org.hibernate.cfg.Configuration getConfigurationInputStream INFO: Configuration resource: /hibernate.cfg.xml Oct 25, 2009 2:44:06 AM org.hibernate.cfg.Configuration addResource INFO: Reading mappings from resource : hibernate/Tbluser.hbm.xml Oct 25, 2009 2:44:06 AM org.hibernate.util.XMLHelper$ErrorLogger error SEVERE: Error parsing XML: XML InputStream(1) Document is invalid: no grammar found. Oct 25, 2009 2:44:06 AM org.hibernate.util.XMLHelper$ErrorLogger error SEVERE: Error parsing XML: XML InputStream(1) Document root element "hibernate-mapping", must match DOCTYPE root "null". Exception in thread "main" org.hibernate.InvalidMappingException: Could not parse mapping document from resource hibernate/Tbluser.hbm.xml at org.hibernate.cfg.Configuration.addResource(Configuration.java:569) at org.hibernate.cfg.Configuration.parseMappingElement(Configuration.java:1587) at org.hibernate.cfg.Configuration.parseSessionFactory(Configuration.java:1555) at org.hibernate.cfg.Configuration.doConfigure(Configuration.java:1534) at org.hibernate.cfg.Configuration.doConfigure(Configuration.java:1508) at org.hibernate.cfg.Configuration.configure(Configuration.java:1428) at org.hibernate.cfg.Configuration.configure(Configuration.java:1414) at hibernate.CreateTest.main(CreateTest.java:22) Caused by: org.hibernate.InvalidMappingException: Could not parse mapping document from invalid mapping at org.hibernate.cfg.Configuration.addInputStream(Configuration.java:502) at org.hibernate.cfg.Configuration.addResource(Configuration.java:566) ... 7 more Caused by: org.xml.sax.SAXParseException: Document is invalid: no grammar found. at com.sun.org.apache.xerces.internal.util.ErrorHandlerWrapper.createSAXParseException(ErrorHandlerWrapper.java:195) at com.sun.org.apache.xerces.internal.util.ErrorHandlerWrapper.error(ErrorHandlerWrapper.java:131) at com.sun.org.apache.xerces.internal.impl.XMLErrorReporter.reportError(XMLErrorReporter.java:384) at com.sun.org.apache.xerces.internal.impl.XMLErrorReporter.reportError(XMLErrorReporter.java:318) at com.sun.org.apache.xerces.internal.impl.XMLNSDocumentScannerImpl.scanStartElement(XMLNSDocumentScannerImpl.java:250) at com.sun.org.apache.xerces.internal.impl.XMLNSDocumentScannerImpl$NSContentDriver.scanRootElementHook(XMLNSDocumentScannerImpl.java:626) at com.sun.org.apache.xerces.internal.impl.XMLDocumentFragmentScannerImpl$FragmentContentDriver.next(XMLDocumentFragmentScannerImpl.java:3095) at com.sun.org.apache.xerces.internal.impl.XMLDocumentScannerImpl$PrologDriver.next(XMLDocumentScannerImpl.java:921) at com.sun.org.apache.xerces.internal.impl.XMLDocumentScannerImpl.next(XMLDocumentScannerImpl.java:648) at com.sun.org.apache.xerces.internal.impl.XMLNSDocumentScannerImpl.next(XMLNSDocumentScannerImpl.java:140) at com.sun.org.apache.xerces.internal.impl.XMLDocumentFragmentScannerImpl.scanDocument(XMLDocumentFragmentScannerImpl.java:510) at com.sun.org.apache.xerces.internal.parsers.XML11Configuration.parse(XML11Configuration.java:807) at com.sun.org.apache.xerces.internal.parsers.XML11Configuration.parse(XML11Configuration.java:737) at com.sun.org.apache.xerces.internal.parsers.XMLParser.parse(XMLParser.java:107) at com.sun.org.apache.xerces.internal.parsers.AbstractSAXParser.parse(AbstractSAXParser.java:1205) at com.sun.org.apache.xerces.internal.jaxp.SAXParserImpl$JAXPSAXParser.parse(SAXParserImpl.java:522) at org.dom4j.io.SAXReader.read(SAXReader.java:465) at org.hibernate.cfg.Configuration.addInputStream(Configuration.java:499) ... 8 more Java Result: 1 BUILD SUCCESSFUL (total time: 1 second) hibernate.cfg.xml <?xml version="1.0" encoding="UTF-8"?> <!DOCTYPE hibernate-configuration PUBLIC "-//Hibernate/Hibernate Configuration DTD 3.0//EN" "http://hibernate.sourceforge.net/hibernate-configuration-3.0.dtd"> <hibernate-configuration> <session-factory> <property name="hibernate.dialect">org.hibernate.dialect.MySQLDialect</property> <property name="hibernate.connection.driver_class">com.mysql.jdbc.Driver</property> <property name="hibernate.connection.url">jdbc:mysql://localhost:3306/hibernate</property> <property name="hibernate.connection.username">root</property> </session-factory> </hibernate-configuration> Tbluser.hbm.xml <?xml version="1.0"?> <!DOCTYPE hibernate-mapping PUBLIC "-//Hibernate/Hibernate Mapping DTD 3.0//EN" "http://hibernate.sourceforge.net/hibernate-mapping-3.0.dtd"> <!-- Generated Oct 25, 2009 2:37:30 AM by Hibernate Tools 3.2.1.GA --> <hibernate-mapping> <class name="hibernate.Tbluser" table="tbluser" catalog="hibernate"> <id name="userId" type="java.lang.Integer"> <column name="userID" /> <generator class="identity" /> </id> <property name="username" type="string"> <column name="username" length="50" /> </property> <property name="password" type="string"> <column name="password" length="50" /> </property> <property name="email" type="string"> <column name="email" length="50" /> </property> <property name="phone" type="string"> <column name="phone" length="50" /> </property> <property name="groupId" type="java.lang.Integer"> <column name="groupID" /> </property> </class> </hibernate-mapping> Tbluser.java package hibernate; // Generated Oct 25, 2009 2:37:30 AM by Hibernate Tools 3.2.1.GA /** * Tbluser generated by hbm2java */ public class Tbluser implements java.io.Serializable { private Integer userId; private String username; private String password; private String email; private String phone; private Integer groupId; public Tbluser() { } public Tbluser(String username, String password, String email, String phone, Integer groupId) { this.username = username; this.password = password; this.email = email; this.phone = phone; this.groupId = groupId; } public Integer getUserId() { return this.userId; } public void setUserId(Integer userId) { this.userId = userId; } public String getUsername() { return this.username; } public void setUsername(String username) { this.username = username; } public String getPassword() { return this.password; } public void setPassword(String password) { this.password = password; } public String getEmail() { return this.email; } public void setEmail(String email) { this.email = email; } public String getPhone() { return this.phone; } public void setPhone(String phone) { this.phone = phone; } public Integer getGroupId() { return this.groupId; } public void setGroupId(Integer groupId) { this.groupId = groupId; } }

    Read the article

  • .NET file Decryption - Bad Data

    - by Jon
    I am in the process of rewriting an old application. The old app stored data in a scoreboard file that was encrypted with the following code: private const String SSecretKey = @"?B?n?Mj?"; public DataTable GetScoreboardFromFile() { FileInfo f = new FileInfo(scoreBoardLocation); if (!f.Exists) { return setupNewScoreBoard(); } DESCryptoServiceProvider DES = new DESCryptoServiceProvider(); //A 64 bit key and IV is required for this provider. //Set secret key For DES algorithm. DES.Key = ASCIIEncoding.ASCII.GetBytes(SSecretKey); //Set initialization vector. DES.IV = ASCIIEncoding.ASCII.GetBytes(SSecretKey); //Create a file stream to read the encrypted file back. FileStream fsread = new FileStream(scoreBoardLocation, FileMode.Open, FileAccess.Read); //Create a DES decryptor from the DES instance. ICryptoTransform desdecrypt = DES.CreateDecryptor(); //Create crypto stream set to read and do a //DES decryption transform on incoming bytes. CryptoStream cryptostreamDecr = new CryptoStream(fsread, desdecrypt, CryptoStreamMode.Read); DataTable dTable = new DataTable("scoreboard"); dTable.ReadXml(new StreamReader(cryptostreamDecr)); cryptostreamDecr.Close(); fsread.Close(); return dTable; } This works fine. I have copied the code into my new app so that I can create a legacy loader and convert the data into the new format. The problem is I get a "Bad Data" error: System.Security.Cryptography.CryptographicException was unhandled Message="Bad Data.\r\n" Source="mscorlib" The error fires at this line: dTable.ReadXml(new StreamReader(cryptostreamDecr)); The encrypted file was created today on the same machine with the old code. I guess that maybe the encryption / decryption process uses the application name / file or something and therefore means I can not open it. Does anyone have an idea as to: A) Be able explain why this isn't working? B) Offer a solution that would allow me to be able to open files that were created with the legacy application and be able to convert them please? Here is the whole class that deals with loading and saving the scoreboard: using System; using System.Collections.Generic; using System.Text; using System.Security.Cryptography; using System.Runtime.InteropServices; using System.IO; using System.Data; using System.Xml; using System.Threading; namespace JawBreaker { [Serializable] class ScoreBoardLoader { private Jawbreaker jawbreaker; private String sSecretKey = @"?B?n?Mj?"; private String scoreBoardFileLocation = ""; private bool keepScoreBoardUpdated = true; private int intTimer = 180000; public ScoreBoardLoader(Jawbreaker jawbreaker, String scoreBoardFileLocation) { this.jawbreaker = jawbreaker; this.scoreBoardFileLocation = scoreBoardFileLocation; } // Call this function to remove the key from memory after use for security [System.Runtime.InteropServices.DllImport("KERNEL32.DLL", EntryPoint = "RtlZeroMemory")] public static extern bool ZeroMemory(IntPtr Destination, int Length); // Function to Generate a 64 bits Key. private string GenerateKey() { // Create an instance of Symetric Algorithm. Key and IV is generated automatically. DESCryptoServiceProvider desCrypto = (DESCryptoServiceProvider)DESCryptoServiceProvider.Create(); // Use the Automatically generated key for Encryption. return ASCIIEncoding.ASCII.GetString(desCrypto.Key); } public void writeScoreboardToFile() { DataTable tempScoreBoard = getScoreboardFromFile(); //add in the new scores to the end of the file. for (int i = 0; i < jawbreaker.Scoreboard.Rows.Count; i++) { DataRow row = tempScoreBoard.NewRow(); row.ItemArray = jawbreaker.Scoreboard.Rows[i].ItemArray; tempScoreBoard.Rows.Add(row); } //before it is written back to the file make sure we update the sync info if (jawbreaker.SyncScoreboard) { //connect to webservice, login and update all the scores that have not been synced. for (int i = 0; i < tempScoreBoard.Rows.Count; i++) { try { //check to see if that row has been synced to the server if (!Boolean.Parse(tempScoreBoard.Rows[i].ItemArray[7].ToString())) { //sync info to server //update the row to say that it has been updated object[] tempArray = tempScoreBoard.Rows[i].ItemArray; tempArray[7] = true; tempScoreBoard.Rows[i].ItemArray = tempArray; tempScoreBoard.AcceptChanges(); } } catch (Exception ex) { jawbreaker.writeErrorToLog("ERROR OCCURED DURING SYNC TO SERVER UPDATE: " + ex.Message); } } } FileStream fsEncrypted = new FileStream(scoreBoardFileLocation, FileMode.Create, FileAccess.Write); DESCryptoServiceProvider DES = new DESCryptoServiceProvider(); DES.Key = ASCIIEncoding.ASCII.GetBytes(sSecretKey); DES.IV = ASCIIEncoding.ASCII.GetBytes(sSecretKey); ICryptoTransform desencrypt = DES.CreateEncryptor(); CryptoStream cryptostream = new CryptoStream(fsEncrypted, desencrypt, CryptoStreamMode.Write); MemoryStream ms = new MemoryStream(); tempScoreBoard.WriteXml(ms, XmlWriteMode.WriteSchema); ms.Position = 0; byte[] bitarray = new byte[ms.Length]; ms.Read(bitarray, 0, bitarray.Length); cryptostream.Write(bitarray, 0, bitarray.Length); cryptostream.Close(); ms.Close(); //now the scores have been added to the file remove them from the datatable jawbreaker.Scoreboard.Rows.Clear(); } public void startPeriodicScoreboardWriteToFile() { while (keepScoreBoardUpdated) { //three minute sleep. Thread.Sleep(intTimer); writeScoreboardToFile(); } } public void stopPeriodicScoreboardWriteToFile() { keepScoreBoardUpdated = false; } public int IntTimer { get { return intTimer; } set { intTimer = value; } } public DataTable getScoreboardFromFile() { FileInfo f = new FileInfo(scoreBoardFileLocation); if (!f.Exists) { jawbreaker.writeInfoToLog("Scoreboard not there so creating new one"); return setupNewScoreBoard(); } else { DESCryptoServiceProvider DES = new DESCryptoServiceProvider(); //A 64 bit key and IV is required for this provider. //Set secret key For DES algorithm. DES.Key = ASCIIEncoding.ASCII.GetBytes(sSecretKey); //Set initialization vector. DES.IV = ASCIIEncoding.ASCII.GetBytes(sSecretKey); //Create a file stream to read the encrypted file back. FileStream fsread = new FileStream(scoreBoardFileLocation, FileMode.Open, FileAccess.Read); //Create a DES decryptor from the DES instance. ICryptoTransform desdecrypt = DES.CreateDecryptor(); //Create crypto stream set to read and do a //DES decryption transform on incoming bytes. CryptoStream cryptostreamDecr = new CryptoStream(fsread, desdecrypt, CryptoStreamMode.Read); DataTable dTable = new DataTable("scoreboard"); dTable.ReadXml(new StreamReader(cryptostreamDecr)); cryptostreamDecr.Close(); fsread.Close(); return dTable; } } public DataTable setupNewScoreBoard() { //scoreboard info into dataset DataTable scoreboard = new DataTable("scoreboard"); scoreboard.Columns.Add(new DataColumn("playername", System.Type.GetType("System.String"))); scoreboard.Columns.Add(new DataColumn("score", System.Type.GetType("System.Int32"))); scoreboard.Columns.Add(new DataColumn("ballnumber", System.Type.GetType("System.Int32"))); scoreboard.Columns.Add(new DataColumn("xsize", System.Type.GetType("System.Int32"))); scoreboard.Columns.Add(new DataColumn("ysize", System.Type.GetType("System.Int32"))); scoreboard.Columns.Add(new DataColumn("gametype", System.Type.GetType("System.String"))); scoreboard.Columns.Add(new DataColumn("date", System.Type.GetType("System.DateTime"))); scoreboard.Columns.Add(new DataColumn("synced", System.Type.GetType("System.Boolean"))); scoreboard.AcceptChanges(); return scoreboard; } private void Run() { // For additional security Pin the key. GCHandle gch = GCHandle.Alloc(sSecretKey, GCHandleType.Pinned); // Remove the Key from memory. ZeroMemory(gch.AddrOfPinnedObject(), sSecretKey.Length * 2); gch.Free(); } } }

    Read the article

< Previous Page | 21 22 23 24 25 26  | Next Page >